Sie sind auf Seite 1von 45

Abdulrahman Bashire 12/15/2011 ABDULRAHMAN BASHIRE

Children hospital Benghazi

1) Normal baby of 12 months, born at term would be expected to be able to:A. Get into sitting position B. Build tower of four bricks C. Have hearing reliably assessed using distraction hearing test. D. Feed with spoon E. Point to eyes, nose & mouth Ans:- A C 2) Regarding Gross Motor Development, a normal child would be expected to:A. Roll over by 2 months B. Ride a tricycle at 2 years C. Kick a ball at 2 years D. Cruise by 1 year E. Not run until 2 years Ans:- C D Explanation:- Importantly, a child wont roll at 2 months as spuriously suggested by perpetrators in cases of non accidental injuries. This milestone is at around 4 months. Cruising is moving around the round by holding on to furniture before independent walking begins and can be expected by 1 year. Running is usually achieved by 18 months. A child should kick as well as jump at 2 years. Riding a tricycle is expected at the 3 year . 3) The following statements are true concerning primitive reflexes:A. It is normal for the Moro reflex to be present at 8 month of age B. The palmar grasp reflex usually disappear by three month of age C. Persistence of asymmetric tonic neck reflex is an early sign of C palsy D. The parachute reflex is present at birth E. The plantar reflex is normally flexor by 4 weeks Ans:- B C 4) The following is true about reflexes present in normal neonate:A. Palmar grasp start to appear from 34 week gestation B. Brisk deep tendon reflexes are present from birth C. More than 10 beat of clonus is abnormal D. An extensor plantar reflex is abnormal E. Rooting reflexes occur after 28 weeks gestation Ans:- B Palmar grasp start to appear from 28 week gestation onwards, deep tendon reflexes tend to be brisk initially, upto 10-20 beats of clonus are normal , Babiniski reflex is usually extensor in infancy. Rooting reflex is usually present from 32 weeks. 5) Normal child of 18 months who would be expected to be able to do the following:A. Climb the stair in an adult fashion B. Draw circle C. Point to eyes, nose , and mouth when instructed to do so D. Speak in short sentences E. Eat with spoon & fork Ans:- C

Abdulrahman Bashire 6) A normal two year old child would be A. Drink from a cup. B. Build a tower of 5 blocks. C. Copy a circle. D. Kick a ball. E. Hop on one foot. 7)

2 expected to:

Children hospital Benghazi

Ans:-ABD The following abnormal finding in 7 month old infant:A. Extensor plantar reflex B. Moro reflex C. Asymmetrical tonic neck reflex D. Obligatory palmar grasp E. Crossed adducor spread

Ans:- BCD plantar reflex are normally extensor in the newborn and this usually persist for most of the first year of life ( found in 75% of normal infants at 12 months of age) , Moro reflex& Asymmetrical tonic neck reflex are abnormal if they persist beyond 6 months of age ,most babies have lost their Moro reflex by 3 month of age. Obligatory palmar grasp which is present at birth normally disappear by 2 months of age & is replaced by voluntary grasping from about 4 months of age, Elicitation of the knee tendon jerk at birth usually causes crossed adductor spasm , this normally disappears by 8 month of age. 8) The following statements are true:A. A moro reflex with the fists remaining clenched is normal. B. The plantar grasp persists longer than the palmar grasp C. Deep tendon reflexes are difficult to elicit at birth D. An extensor Babinski response is normal in the first year E. A newborn baby from 32 weeks gestation will turn towards a light Ans:- B D E Explanation:- Most primitive reflexes disappear by 6 months of age. Exceptions are the plantar reflex (9 months), asymmetric tonic neck reflex (9 months), stepping reflex (11 months). The moro reflex consists of arm extension followed by flexion with extension of the fingers, and usually flexion of the thighs at the hips. A moro reflex with the fists remaining clenched is abnormal. Deep tendon reflexes are present and usually brisk at birth. The Babinski response is usually extensor until the infant begins to walk (usually from 1 year). A newborn baby from 32 weeks gestation with a normally functioning visual cortex will turn the head towards a light. 9) At 5 months of age normal baby :A. Can sit unsupported B. Can roll from front to back C. Exhibits the Moro reflex D. Uses pincer grip E. Has head lag present Ans:- A B 10) A normally developing 18-month-old child: A. Should recognise his/her name B. Can easily distinguish primary colours by name C. Plays in parallel with his/her peers D. Should imitate observed actions E. Should be able to copy a vertical line Ans:- A C D Explanation:- 18-month-old child - normal developmental milestones Gross motor development: * walks well with feet only slightly apart - some children are not walking yet, but are bottom shuffling 2

Abdulrahman Bashire 3 Children hospital Benghazi * pulls a toy as he walks * runs stiffly * climbs stairs holding on * throws a ball without falling Fine motor development: * tower of three cubes * scribbles * turns pages of a book two or three at a time * can use a spoon without rotation * can take off gloves and socks and unzip fasteners Personal and social development: - General understanding * points to own or dolls nose, hair, eyes on request - 2-3 parts of the body * imitative play * points to named pictures - Speech and language * 6-20 recognisable words * names 1-2 simple objects - Social behaviour * feeding - can use spoon without rotating it * drinks from a cup * may be dry by day (Forfar and Arneil's textbook of Paediatrics) 11) Normal development include:A. The presence of the Moro reflex at 3 months of age B. Being able to hop on one foot at age of 3 years C. The presence of downward parachute by the age of 3 months D. The presence of forward parachute by the age of 7 months E. Hands being brought together in finger play by the age of 3 months Ans:- A D E The presence of downward parachute by the age of 5 months , The child can usually stand on one preferred foot momentarily when shown and can hop on one foot at age of 4 years. 12) These statements relating to normal development are corrects:A. 2 year old can turn pages one at time B. At 15 months child can build tower of six cubes C. Squint at 6 months is definitely abnormal D. One year old can retrieve toy hidden before his or her eyes under cup or cushion E. At 5 year child can copy square but not triangle Ans:- A C D E 13) Toe-walking in an 18 month old child :A. Is always abnormal B. Spinal cord imaging may be indicated C. Serum creatine kinase should be measured D. Does not need to be investigated in a girl E. Does not need to be investigated in a girl Ans:-BCE Explanation:- Pathological causes of toe walking are i) neuromuscular disorders e.g. Duchenne muscular (where initially due to proximal weakness and pelvic instability shifting the centre of gravity forward and later to tightness of the tendo achilles) dystrophy. Creatine kinase is a muscle enzyme and is increased in conditions with increased muscle turnover e.g. muscular dystrophies or myopathies. ii) upper motor neurone lesions (i.e. in brain and spinal cord) due to extensor hypertonus and tightness of 3

Abdulrahman Bashire 4 tendo achilles. Toe walking may be a feature of diplegic cerebral palsy. Toe walking may be normal but this is a diagnosis of exclusion.

Children hospital Benghazi

14) Regarding child development: A. Sense of taste is present from birth. B. 25% of children are left handed. C. Left handedness is more common in exprem babies without cerebral palsy D. Left handed children are more likely to have un intentional injuries. E. Prevalence rate of thumb-sucking in children are universal across races. Ans:-ACD Explanation:- Sense of taste is prersent from birth. Infants show preference for sweet over salty or plain water. 7-10% of children are left handed. This is a surprise as animals e.g. cats & dogs are equally left or right paired. Up to 25% of exprem babies without cerebral palsy are left handed but the reason is unclear. It is thought to be due to antecedent brain injury. Left handed children are more likely to have un intentional injuries as well as a higher likelihood of developmental disorders e.g.dyslexia and autism. Prevalence rates of thum-sucking in children vary from 45% of US children to 1% in eskimos. It tends to be less where children are carried by parents. 15) To be effective for preventing meningomyelocele, administration of folic acid needs to begin: A. By 3 months of gestation B. At the first missed period C. Before conception D. By 30 days of gestation E. At 3 months prior to delivery Ans:- C Explanation: Folic acid supplementation has been a major public health success in reducing the incidence of neural tube defects in the United States. Supplementation also reduces the recurrence rate of neural tube defects in families with a previously affected child. 16) A newborn with myelomeningocele is being evaluated, and the parents ask about the likelihood of the development of hydrocephalus. Which of the following best describes the risk of hydrocephalus in this situation? A. The risk increases with higher spine lesions B. The risk decreases with higher spine lesions C. The risk is unknown at birth D. The risk is less than 25% E. The risk is related to the cause of the malformation Ans:- A Explanation: Hydrocephalus is quite common in children with a myelomeningocele and must be evaluated with a head ultrasound study or CT. 17) Feature of meningomyelocele suggestive of poor prognosis include:A. Hydrocephalus after surgery B. Thoracic lesion C. Associated visceral abnormality D. Associated hemivertebrae E. Neuropathic bladder Ans:-B C E Poor prognosis associated with meningomyelocele are:- thoracolumbar lesion, sever paraplegia below level of L3, kyphosis, hydrocephalus before surgical closure, intracranial birth injury and sever congenital deformities, the majority of infants develop hydrocephalous after closure of the lesion 18) Dandy -Walker syndrome:4

Abdulrahman Bashire 5 A. Classically present with dilated third B. Often clinically demonstrate cerebellar ataxia C. Shouldn't be shunted D. It isn't associated with mental retardation E. May be associated with an occipital encephalocele

Children hospital Benghazi and lateral ventricles

Ans:- B E The Dandy-Walker malformation consists of a cystic expansion of the 4th ventricle in the posterior fossa and midline cerebellar hypoplasia, ( dilated fourth ventricle), there may be an associated occipital encephalocele. Obstructive hydrocephalus & cerebellar ataxia are often feature of presentation. Mental retardation which can be severe occur in about 70% of cases, shunt are often required not only to the lateral ventricle but also to the cystic fourth ventricle itself . 19) Sturge-Weber syndrome. A. Occurs in 50% of children with port wine stains in the ophthalmic division of the trigeminal nerve B. Results in hemiplagia on the ipsilateral side of the vascular lesion. C. Is associated with cataracts D. Typically are associated with slite-like ventricles on CT E. Is associated with a SXR showing a pepperpot skull. Ans:- ALL FALSE Sturge-Weber syndrome is a rare (1 in 50.000) sporadic combination of facial naevus &seizures, hemiparesis and often low IQ. 1- Occurs in 8% with unilateral lesion & 33% with bilateral lesion, diagnosis is by MRI. 2- Refractory focal tonic-clonic fits leads to a progressive hemiparesis contralaterally. 3- Infantile glaucoma (buphthalamos) commonly occurs,Ipsilaterally. 4- The CT shows unilateral dilated ventricle, calcification & cerebral atrophy. The SXR shows the intracranial calcification in the occipitoparietal lesions as tram-line or railroad calcification 20) Recognized features of Tuberous sclerosis include:A. Caf-au-lait patches B. Axillary freckling C. Adenoma sebaceum ( facial angiofibroma) D. Gyral calcification E. Ungual fibroma Ans:- CE Gyral calcification occur in Sturge-Weber syndrome , 21) Feature of Tuberous sclerosis include:A. 50% recurrence risk in offspring B. Adenoma sebaceum C. Hypsarrhthmic changes on the EEG D. Good response of seizure to treatment with Vigabatrin E. The frequent occurrence of malignant tumors Ans:-ABCD 22) Concerning neurofibromatosis:A. Acoustic neuroma are present in type B. Maternal folate deficiency is risk fector C. Optic nerve glioma are seen D. Is usually associated with intellectual impairment E. Type is commoner than type Ans:- C 5

Abdulrahman Bashire 6 23) In von Recklinghausen's disease:A. As few as three caf - au lait spots are diagnostic B. Axillary freckling support the diagnosis C. There is recognized association with scoliosis D. There is an autosomal recessive form of inheritance E. Fibromas usually only appear at puberty

Children hospital Benghazi

Ans:-BCE 24) Pregnant woman with neurofibromatosis has what risk of an affected fetus:A. 0% B. 10% C. 25% D. 50% E. 100% Ans:- D 25) Neurofibromatosis type 1, an autosomal dominant disorder (gene on chromosome 17), is defined by six or more caf-au-lait macules greater than 5 mm in diameter in prepubertal children or greater than 15 mm in postpubertal children plus at least one of the following except: A. Axillary or inguinal freckling B. Lisch nodules of the iris C. Two or more neurofibromas or one plexiform neurofibroma D. Ash leaf macule E. Osseous lesions (sphenoid dysplasia, scoliosis) F. Optic gliomas G. An affected first-degree relative Ans:- D Explanation: The ash leaf-shaped hypopigmented macule is most typical of tuberous sclerosis (being present in over 90% of affected children), another autosomal dominant disorder. 26) A 19-yr-old girl presents with headache, unsteadiness, and poor hearing that has worsened over the past 5 yr. Her father's medical history includes some type of brain surgery, and he has been deaf since the age of 35 yr. The most likely diagnosis is: A. Neurofibromatosis type 2 B. Optic glioma C. Neurofibromatosis type 1 D. Tuberous sclerosis E. Late-onset congenital deafness Ans:- A Explanation: NF2 accounts for 10% of all NF cases, has distinctive chromosomal sites, and is characterized by bilateral acoustic neuromas. Caf-au-lait macules may not be present. 27) Feature of neurofibromatosis type include:A. Pheochromocytoma B. Optic nerve glioma C. Retinal phakomata D. Hamatomata E. Duodenal carcinoid Ans:- A B E 28) A 6-yr-old child with neurofibromatosis (NF1) is found to have an optic glioma on a routine MRI study, confined to the right optic nerve. Findings on the neurologic, physical, and retinal examinations are normal. The visual acuity is 20/20 bilaterally, uncorrected. Which of the following is the correct management? A. Surgical removal of the tumor 6

Abdulrahman Bashire 7 B. MRI of the optic nerve every 3 m C. Chemotherapy D. Annual examination by a pediatric ophthalmologis E. Radiation therapy limited to the right optic nerve

Children hospital Benghazi

Ans:- D Explanation: Patients with NF1 and an optic glioma as in this case can be observed over time. In many situations the tumor is slow growing and produces few problems. 29) Macrocephaly :A. Is defined as Head circumference > 1 SD above mean B. Is associated with high intelligence C. Is associated with Cornelia de Lange syndrome D. Can be inherited E. Is associated with neurofibromatosis Ans:- D E Explanation:macrocephaly:Head circumference (HC) > 2SDs above mean microcephaly: HC > 2SDs below mean Causes of Macrocephaly: Causes of Microcephaly: Familial Familial Achondroplasia Congenital infection Canavans disease Fetal alcohol syndrome Neurofibromatosis Cornelia de Lange syndrome Osteopetrosis Angelman syndrome Tay Sachs disease Edwards and Patau syndrome Sotos syndrome Rubenstein Taybi syndrome. 30) Macrocephaly is found in:A. Edward 's syndrome B. Tay-Sach's syndrome C. Achondroplasia D. Cornelia de Lange syndrome E. Neurofibromatosis Ans:- B C E 31) Recognized causes of macrocephaly include:A. Secal syndrome B. Neurofibromatosis type C. Congenital CMV infection D. Rett syndrome E. Communicating hydrocephalus Ans:- B E 32) Causes of megalocephaly include all of the following except: A. Thalassemia B. Chronic subdural effusions C. Hydrocephalus D. Canavan disease E. Congenital CMV infection F. Familial factors Ans:- E Explanation: Congenital cytomegalovirus (CMV) infection usually causes microcephalus, not macrocephalus. Expansion of the bone marrow (hemolytic anemias), storage diseases (lysosomal, leukodystrophies), excessive cerebrospinal fluid (CSF) (hydrocephalus), intracranial bleeding (subdural), and familial factors contribute to megalocephaly. 7

Abdulrahman Bashire

Children hospital Benghazi

33) Head circumference above the 97th centile:A. May be normal in child whose hight & weight are on the 50th centile B. Is characteristic of the congenital rubella syndrome C. If found in cojunction with fundal hemorrhage in 6 month old infant is likely to be secondary to Haemophilus meningitis D. May be associated with the Arnold Chiari malformation E. May be feature of cerebral gigantism ( soto syndrome) Ans:-ADE Sotos syndrome characterized by large head, somatic gigantism, abnormal facies & mental retardation 34) Problem characteristic of child with spina bifida include:A. Kyphoscliosis B. Delayed puberty C. Hydrocephalus D. Incontinence E. Elective mutism Ans:- ACD Child with spina bifida may have premature puberty. They also have coktail party speech instead of mutism 35) The following condition are associated with hydrocephalus:A. Anaemia B. Klippel-Feil syndrome C. Choroids plexus papiloma D. Dandy Walker malformation E. Mucopolysacchaides Ans:-BCD 36) Febrile convulsion:A. Occur in 15% of the population B. May occur in neonatal period C. May be prevented by using phenytoin D. May be familial E. Recur in about 3% of children Ans:- D 37) Risk factors for simple febrile convulsions include:A. Age less than 6 months B. Family history of epilepsy C. Family history of febrile convulsion D. Past history of urinary tract infection E. Previous febrile convulsion Ans:- C E 38) Which of the following isn't typical of simple febrile convulsion:A. Age of onset between 6 months and 6 years B. Seizure lasting less than 15 minute C. Positive family history D. Todd's paralysis E. Seizure more likely to occur with rapid raise in temperature Ans:- D Todd's paralysis or postseizure abnormal neurologic sign aren't part of the picture 39) When considering febrile convulsion:A. The incidence of febrile convulsion in infancy is 0.01% B. The age of greatest susceptibility is between 1 and 6 month of age 8

Abdulrahman Bashire 9 Children hospital Benghazi C. Focus of infection is not required as long as temperature is above 40 C D. The longer convulsion last the less likely is the risk of recurrence E. Recurrence rate are of the order of 0.3% Ans: - ALL FALSE The incidence of febrile convulsion in infancy is 5-7% 40) Simple febrile seizures: A. Most commonly occur between the ages of 6 months and 5 years. B. The resting EEG is typically abnormal C. Are associated with structural brain abnormalities. D. Are associated with a very slight increase in adult seizure frequency. E. Seizures following perinatal injury may be accompanied by fever. Ans:- A D? E Explanation:- The peak incidence of febrile seizures is between 9 and 20 months. In most cases the EEG following a simple febrile seizure is entirly normal. Where febrile seizure are defined as simple they occur in the absence of underlying structural abnormalities. The increased rate of epilepsy in adults following febrile seizures is approximately 2-4%. Seizures following perinatal injury may be associated with a pyrexia, although this is not the primary event. 41) Febrile convulsion:A. Characteristically occur on the second day of feverish episodes B. Can usefully be treated by intramuscular diazepam C. Usefully be treated by sodium valproate per rectum D. Are best prevented by long-term phenytoin therapy E. Have peak incidence between the age of 3-12 month Ans:- ALL FALSE Febrile convulsion characteristically occur during the initial period of the onset of fever, and are surprisingly uncommon on subsequent days. This feature makes them difficult to anticipate and sometimes means that the presence of significant pyrexia is appreciated only after the convulsion has occurred. 42) A 3 year old girl presents with her first febrile convulsion. The following information should be given to her parents:A. Febrile convulsion occur in 5% of children B. She has 5% chance of developing idiopathic epilepsy C. She has 25% chance of having further febrile convulsion D. Phenobarbitone will prevent any further seizures occurring E. Preschool booster vaccination should not given Ans:- A C 43) The likelihood of child with febrile convulsion developing later epilepsy is influenced by:A. The presence of family history of febrile convulsions B. History of partial febrile convulsions C. The onset of febrile convulsion before 12 months D. The number of seizures occurring within single febrile illness E. The duration of the febrile seizures Ans:- B D E The onset of febrile convulsion before 6 months would carry the 10% risk mentioned above ,but before 12 months is unremarkable 44) Which of the following may be mistaken for childhood or infantile seizures;A. Reflex anoxic episodes B. Romano Ward syndrome C. Night terrors D. Masturbation 9

Abdulrahman Bashire E. Breath holding episodes

10

Children hospital Benghazi Ans:- ABCDE

45) Concerning epilepsy:A. In complex seizure consciousness is lost B. Simple refers to short duration C. Partial seizure begin focally D. Epilepsy that is difficult to control is classified as symptomatic E. An aura is necessary to make diagnosis Ans:- AC 46) 9 year old child has first single generalized tonic clonic seizure:A. The risk of second seizure is approximately 80% B. The child should be cautioned against cyclic in traffic C. CT scan should be performed D. EEG may show generalized 3 Hz spike and waves activity upon hyperventilation E. Partial seizures around the mouth could be helpful diagnostically Ans;-BE 47) Regarding epilepsy in children:A. As group, children with epilepsy show high prevalence of psychiatric disorders B. Seizures are characteristic of disease affecting cerebral white matter C. The incidence of epilepsy is higher in the neonatal period than at any later age D. Half of all cases of epilepsy have presented by the age of 8 E. Idiopathic neonatal seizures have reasonable good prognosis Ans:-ACE Children with uncomplicated epilepsy have 4 fold higher risk of psychiatric disorder, rising to 10 fold if there was an associated neurological disorder Seizure are result of cortical ( gray matter ) disease ,in general ,white matter disease tend to present with motor( pyramidal or cerebral ) sign. Only 50% of cases of epilepsy have presented by the age of 15-16. Natural neonatal seizure due to hypoxic-ischemic encephalopathy, neonatal infection or congenital anomalies have far worse prognosis 48) Characteristics of simple partial seizures include all of the following except: A. Loss of consciousness B. Versive seizures C. Duration of 10-20 sec D. No postictal state E. Abnormal EEG findings Ans:- A Explanation: There is never a loss of consciousness in simple partial seizures. Actually, some patients may talk to those around them during the event. 49) Concerning complex partial seizure ( temporal lobe epilepsy ):A. Consciossness is impaired B. Ethosuximide is the drug of choice C. The EEG show 3/ second spikes D. They can present as drop attacks E. They are the commonest form of childhood epilepsy Ans:-AD 50) Regarding complex partial epilepsy:A. Absence seizures and generalised tonic-clonic seizures may occur B. Consciousness is altered C. Hyperventilation often provokes a complex partial seizure D. EEG changes disappear in sleep 10

Abdulrahman Bashire E. Ethosuximide is an affective

11 treatment

Children hospital Benghazi

Ans:-AB Explanation:- Complex partial seizures, by definition, include altered consciousness. They are focal frontotemporal seizures. However complex partial epilepsy can involve generalised seizures such as absence and tonic-clonic seizures. Sleep often enhances EEG abnormalities. Hyperventilation provokes absence seizures, not complex partial seizures. Carbamazepine and sodium valproate are effective treatments for complex partial seizures. Ethosuximide is only effective in absence seizures. Rasmussen encephalitis is characterized by all of the following except: A. Epilepsia partialis continue B. Onset before age 10 yr C. No sequelae D. Abnormal EEG findings E. Possible role of CMV infection Ans:- C Explanation: Sequelae may include hemiplegia, hemianopia, or aphasia. 51) A 9 yr old girl is seen in clinic. Her teacher at school has reported that she has numerous episodes throughout the day of abruptly staring into space mid-conversation for a few seconds with rapid return of awareness. Which of the following features suggest complex partial rather than absence seizures as the cause of her problem :A. Reporting unusual smells prior to the episodes B. Provocation of the episodes by hyperventilation C. Abnormal features on MRI scanning of the brain D. A good response to anti-epileptic drugs E. A normal EEG Ans:- A C E Explanation:- Complex partial seizures may present with brief periods of reduced awareness. Features which make complex partial seizures, rather than absence seizures likely, include: Later onset, longer duration, the presence of aura and post-ictal phenomena, focal abnormalities or normal EEG (absences tend to have typical 3HZ spike pattern), associated MRI abnormalities (mesial temporal sclerosis, for example) and a poor response to anti-epileptic therapies. 52) Landau-Kleffner syndrome is characterized by all of the following except: A. Onset at age 5 yr B. More common in girls C. Aphasia D. Normal hearing E. Multiple seizure types Ans:- B Explanation: The disorder is more common in boys. The etiology is unknown, and the treatment of choice is valproic acid. 53) A 4-yr-old boy is evaluated for his first generalized tonic-clonic seizure, which lasted 10 min. There is no history of illness or fever, and findings on examination an hour after the seizure are completely normal. The most appropriate management is: A. Begin therapy with carbamazepine B. Order an EEG C. Order a CT scan of the brain D. Order an MRI study of the brain E. Order psychometric testing Ans:- B 11

Abdulrahman Bashire 12 Children hospital Benghazi Explanation: An EEG will help define the presence of a seizure focus and if there is a specific seizure type. A normal EEG may help the pediatric neurologist determine if anticonvulsant therapy is needed. 54) A 15-mo-old girl is evaluated for a 10-min-long generalized seizure associated with a temperature of 40oC. Which of the following factors in the history is most likely to increase the risk of future seizures? A. APGAR score of 3 at 5 min B. Family history of epilepsy C. Clinical evidence of roseola D. Female gender E. Presence of 2 caf-au-lait spots Ans:- B Explanation: Future nonfebrile seizures can be predicted when there is a family history of seizures. Up to 30% of children with febrile seizures have a recurrence when they becom febrile; these are often unpredictable 55) In the assessment and management of epilepsy in childhood:A. Normal EEG will exclude the diagnosis in clinically doubtful case B. I/M diazepam is more effective than rectal diazepam in the treatment of status epilepticus C. Na valproate has been associated with liver damage D. Phenytoin can cause rickets E. Petit mal is characterized by retrograde amnesia Ans:-CD 56) In the management of childhood epilepsy:A. Swimming should be prohibited B. Phenobarbitone may cause gum hyperplasia C. Infantile spasm may respond to ACTH D. Most patient should be educated in special schools E. Anticonvulsant medication should be prescribed for 2-3 years after first non febrile convulsion Ans:-C 57) It has been determined that sodium valproate is the best anticonvulsant medication for a 12-yrold boy with epilepsy. His parents ask about the possibility of fatal liver necrosis if he takes the drug. Which of the following factors markedly increases the risk of this complication? A. Male gender B. Age younger than 2 yr C. Down syndrome D. Family history of epilepsy E. Allergic reactions to phenobarbital Ans:- B Explanation: Young age is a risk factor for hepatic and perhaps pancreatic complications of valproic acid. It is much safer in 12-yr-old children than those younger than 2 yr. 58) The chances of severe life-threatening skin reactions such as Stevens-Johnson syndrome are greatest with which of the following anticonvulsant medications? A. Phenobarbital B. Gabapentin C. Carbamazepine D. Lamotrigine E. Sodium valproate Ans:- D Explanation: In addition, phenobarbital and carbamazepine are also associated with an acute drug hypersensitivity syndrome involving skin, mucous membranes, and the liver. 12

Abdulrahman Bashire

13

Children hospital Benghazi

59) A 3-yr-old boy is being evaluated after an episode at home during which he lost consciousness for 5 min; he was brought to the emergency department an hour later. On examination, which of the following factors is most helpful in distinguishing whether this episode was a seizure or syncope? A. Family history B. Temperature C. Blood pressure D. Level of consciousness E. Size of pupils Ans:- D Explanation: Syncope usually produces a loss of tone and a fall, with rapid recovery of consciousness once the affected person is in a horizontal position. Occasionally there may be a few tonic-clonic jerking movements with syncope, thus confusing the assessment. 60) In the assessment& investigation of case of presumed idiopathic epilepsy:A. Computerized axial tomography is indicated in all cases under 14 years of age B. It may be appropriate to examine both parents under woods' ulteraviolet light C. If randome blood glucose is normal this precludes hypoglycemia as cause D. If the EEG is abnormal this is an indication for treatment following the first convulsion E. History of rapid recovery of normal consciousness favour the diagnosis of faint rather than fit Ans:- B E CT or MRI scanning should be reserved for cases of focal epilepsy or those with clinical evidence of neurological deficit , 61) Benign rolandic epilepsy:A. Represent less than 1% of childhood epilepsy B. Carries poor prognosis C. Abnormal inter-ictal EEG D. Nocturnal generalized seizure can occur E. There is good response of seizure to sodium valproate Ans:-CD inter-ictal EEG show centro-temporal spike 62) Benign rolandic epilepsy :A. Is a common form of partial epilepsy B. Has a poor prognosis C. Involves seizures that often occur at school D. Can be successfully treated with carbamazepine E. Has a typical EEG with a 3 per second spike and wave pattern Ans:-AD Explanation:- Benign rolandic epilepsy- the most common partial epilepsy in childhood. 15-20% of childhood epilepsy. Seizures usually stop in mid teens, may not require treatment. Seizures usually occur in sleep: sensory aura followed by GTC seizures often involving face, arms. EEG shows typical cento-temporal spikes. Treatment with carbamazepine. Excellent prognosis. 63) Rolandic epilepsy:A. Occur exclusively below 2 year of age B. Is life long condition C. Can occur during sleep D. Is associated with learning difficulties E. Has classical CT changes Ans:- C Rolandic epilepsy accounts for up to 16% of all afebrile seizure below 15 years, in the awake state, motor or sensory symptoms may occur involving the tongue , mouth and face speech & swallowing are commonly impaired and the child may be able to understand what is being said to him but be unable to communicate. 13

Abdulrahman Bashire Family history is noted in 13% of patient, the on CT scanning.

14

Children hospital Benghazi intelligence is normal and no abnormality are found

64) Petit mal(absence attack) are usually characterized by all of the following except:A. Attack rarely last more than 5-15 second B. Typically the child abruptly recovers senses C. There is usually no aura D. Frequency may be increased by fatigue, photic stimulation, and emotional stress E. EEG isn't characteristic Ans:- E 65) Absence seizures:A. May occur in the absence of supportive EEG data B. Are rare in the prepubertal age group C. May be progress to generalized seizures D. Are best treated with Vigabatrin E. Usually have an underlying cortical focal lesion Ans:- C 66) In petit mal epilepsy:A. Attack are often regarded as simple day-dreaming by teachers B. Retrograde amnesia is characteristic C. Attack may be precipitated by over breathing D. The drug of first choice is carbamazepine E. Attack virtually never persist into adulthood Ans:-AC There is no retrograde amnesia ,just amnesia for events occurring during the attack 67) Feature of petit mal epilepsy ( typical absence epilepsy of childhood ) include:A. Age less than 2 years B. Characteristic inter-ictal EEG C. Good response to cabamazapine D. Long term remission E. Long term risk of generalized seizures Ans:- D E 68) Lennox-Gastaut syndrome:A. Carries poor prognosis when associated with West's syndrome B. Consists of more than one type of epilepsy C. Is usually associated with normal intelligence quotient D. Responds to monotherapy E. Usually regresses in adolescence Ans:-AB Lennox-Gastaut syndrome consists of several different form of epilepsy( Atypical absence, axial tonoc seizure & drop attacks) .EEG changes of slow spike and wave when awake and 10 Hz burst when asleep and psychomotor retardation. The condition often require combination of anticonvulsant to effect satisfactory treatment. Only 4-6% of cases are self remitting with poor prognostic factors including preceding developmental delay, neurological abnormality or West's syndrome( infantile spasm ) . Early onset such as before 3 years of age, recurrent status epilepticus, poor fit control and slow background activity are also poor prognostic indicators. 69) Concerning infantile spasms:A. They occur in the first tear of life B. EEG finding are non specific C. The symptomatic group has worse prognosis 14

Abdulrahman Bashire

15

Children hospital Benghazi

D. Vigabatrin frequently used as drug of first choice E. 30% of cases are idiopathic Ans:- ACDE 70) Infantile spasm:A. Indicate underlying brain disorder B. Are incompatible with normal development C. Respond to valproate D. Are usually associated with electrolyte abnormalities E. Are usually indicated by hypsarrhythmia on the EEG Ans:- ABE 71) Drugs which are effective in the treatment of infantile spasm include:A. Carbamazapine B. ACTH C. Vigabatrin D. Na valproate E. Clonazepam Ans:-BCDE 72) The following are true of convulsion in the neonatal period:A. Na valproate is the drug of choice B. Convulsion may respond to injection of pyridoxine C. Convulsion may present as apnoeic attack D. The prognosis for normal development is poor the majority of cases E. History of maternal narcotic addiction may be relevant Ans:-BCE 73) Causes of status epilepticus include: A. Binge drinking. B. Sepsis syndrome. C. Amphetamines but not cocaine. D. Brain stem death. E. Suergons knife. Ans:- A B E Explanation:Acute and chronic alcohol ingestion can cause status. * Cocaine is a potent cause of status, and also subarachnoid hemorrhage. * A prerequisite for brain stem death is absence of seizure activity, an EEG is sometimes needed to confirm this. * Surgery can stimulate seizure activity to define the exact location of epilepsy foci, this is one of the indications for awake craniotomies. 74) Definition of status epilepticus: A. Two or more fits within 30 minutes B. Unresponsive patients who have been incontinent for 60 minutes C. Mouth twitching lasting 60 minutes with reduced GCS D. Tonic-clonic seizure lasting 30 minutes with incontinence E. Rapidly repeating tonic-clonic seizure with not returning to full consiousness Ans:- ALL FALSE Explanation:- * Status epilepticus is defined as a condition in which continuous or rapidly repeating tonicclonic seizures persist for 30 minutes or more. * It is often associated with incontinence but this is required for the diagnosis. * As the seizures persist they often become localized, mouth twitching is a common sign of late status, but there are many other diagnosis in a patient with mouth twitching and reduced GCS 15

Abdulrahman Bashire

16

Children hospital Benghazi

75) The following condition are inherited as autosomal dominant:A. Tuberous sclerosis B. Ataxia telangectasia C. Colour blindiness D. Hemophilia E. Myotonic dysentery Ans:- AE 76) In acute encephalitis:A. Epilepsy is recognised sequalae B. Herpes simplex is the commonest cause of sporadic viral encephalitis in the UK C. Abnormal EEG changes may be present early in the disease even if the CT scan is normal D. Acyclovir is an effective treatment for herpes simplex E. Absence of pleocytosis in the CSF excludes the diagnosis Ans:-ABCD Characteristic EEG changes are often only present late on in the disease process but some changes may be present early on , in the presence of normal CT scan. There may be normal CSF on initial examination, with little or no pleocytosis & remarkably normal biochemistry. 77) The following are causes of aseptic meningitisA. Primary HIV infection. B. Vogt-Koyanagi-Harada syndrome. C. Behcets disease. D. Heerfordets syndrome. E. Haemophilus influenzae type b Ans:- A B C Explanation:- Vogt-Koyanagi-Harada syndrome is recurrent inflammation involving cells of neural crest origin, associated with uveomeningo-encephalitis, depigmentation (vitiligo, poliosis) and deafness. Behcets disease can produce a recurrent meningitis and features may become chronic. Heerfordts syndrome occurs in sarcoidosis, which can be a cause of aseptic meningitis, but the syndrome comprises eye and parotid gland involvement only. 78) 15-year-old teenager presented with few haehorrhagic spots, agitation and confusion having been to school. Temp 39, pulse 110 and BP 120/80. The first management step would be :A. Oxygen B. Intravenous fluid C. Throat swab D. Intravenous antibiotic E. Lumbar puncture Ans:- D Explanation:- The history suggets meningococcal sepsis, IV antibiotics must be given immedietly. 79) In Tuberculous meningitis in childhood: A. The Mantoux reaction is negative B. The CSF glucose level is characteristically reduced C. The CSF reaction consist mainly of polymorphs D. The presentation is characteristically subacute E. Rifampicin isn't useful drug for treating this condition Ans:- B D In the early stages the CSF glucose may be normal & cellular reaction of mixture of polymorphs and lymphocytes . However in most cases by the time of diagnosis the CSF glucose will have fallen & the cellular reaction will be mainly lymphocytic. Compared with cases of viral meningitis, the patient will be more ill , the onset more subacute and CSF protein higher in addition to low CSF glucose,if choroidal tubercule are present they are virtually diagnostic . 16

Abdulrahman Bashire

17

Children hospital Benghazi

80) Tuberculous meningitis in childhood A. Usually occur within 6 months of the primary infection B. Is associated with negative mantoux test in over 90% of cases C. Occur in over 50% of cases of military tuberculosis D. Is associated with normal CSF glucose E. May benefit from systemic steroid therapy as an adjuvant to tuberculous drugs Ans:-A C E mantoux test is usually strong positive and is only likely to be diminished or negativein the patient who is extremely ill and cachectic due to late diagnosis. the CSF glucose is characteristically decreased which differentiate this condition from viral meningitis. It is reasonably orthodox to use systemic steroid therapy in the hope of decreaseing eningeal reaction and the chance of adhesions. Cranial nerve palsies and the risk of spinal shock. 81) The following statement are true with respect to viral encephalitis;A. Subacute sclerosing panencephalitis ( SSPE ) is thought to be due to mumps virus B. Focal sign are common in Herpes simplex encephalitis C. Measles encephalitis characteristically affects the cerebellum D. Encephalitis occur in 1: 1000 cases of measles E. Brain biopsy can confirm the diagnosis in Heres simplex encephalitis Ans:- B D E Heres simplex causes sever necrotizing encephalitis which frequently result in focal neurological signs such as hemiplegia. The diagnosis should be suspected if the patient has recent history of Herpetic cold sores. All cases of possible encephalitis should be treated as early as possible with acyclovire as this can improve the prognosis if the encephalitis is due to H. simplex measles encephalitis affect mainly the cerebrum,varicella encephalitis has an especial predilection for the cerebellum, especially the vermis and may present with truncal ataxia. Brain biopsy can confirm the diagnosis in Heres simplex encephalitis by demonsrating characteristic intranuclear inclusion bodies. 82) Cerebral abscess in childhood:A. May be secondary to congenital heart disease in which there is Lt-Rt shunt B. Should be treated with intrathecal amoxicillin C. May result from frontal sinusitis D. May be benefit from treatment with metronidazole E. Should never be treated with chloramphenicol Ans:- C D Chloramphenicol is very useful drug in both cerebral abscess , and pyogenic meningitis, because of it's broad spectrum and excellent ability to cross both the inflamed and noninflamed BBB. 83) The following condition predispose to cerebral abscess formation:A. Skull fracture involving the frontal sinus B. Suppurative lung disease C. Viral meningitis D. Congenital heart disease with Lt Rt shunt E. Chronic middle ear sepsis Ans:-ABE 84) Which of the following CSF laboratory result are correctly paired with their clinical entities:A. Neutrophil count 20.000/mm3 : tuberculous meningitis B. Protein 4 g/l : viral encephalitis C. CSF glucose 0.9 mmol/l : mumps, meningoencephalitis D. Low CSF chloride : tuberculous meningitis E. CSF glucose/blood glucose ratio of 0.4 : bacterial meningitis Ans:- C D E 17

Abdulrahman Bashire 18 Children hospital Benghazi The CSF picture of bacterial meningitis is of high neutrophil count associated with elevated protein level and low glucose concentration withCSF/blood glucose ratio of not less than 0.6.TB meningitis is associated with an elevated lymphocyte count . Massive CSF protein concentration are not usually elevated in viral encephalitis. Low CSF glucose level are unusual in viral meningitis except mumps. 85) Concerning meningococcal disease: A. Is readily distinguished on clinical grounds from other infectious agents by the spreading purpura B. Presents in seasonal epidemics C. Frequently co-presents with meningitis, making LP necessary at presentation D. Mannose binding protein deficiency is a known risk factor for meningococcaemia E. Ciprofloxacin is used for routine contact prophylaxis Ans:- B D Explanation:- Other risk factors include splenectomy, terminal complement deficiencies (C6-C9). Ciprofloxacin is used as an alternative prophylactic agent to rifampicin especially in pregnancy or those taking the oral contraceptive pill. 86) The following are recognized complication of ventriculatrial shunt for treatment of hydrocephalus:A. Meningitis B. Superior vena caval thrombosis C. Secondary Polycythemia D. Pulmonary hypertension E. Intraventicular haemarrhage Ans:- A B D E 87) The following signs can be present in child with Asperger's syndrome:A. Significant language delay B. Stereotypical pattern of behaviour C. Lack of eye to eye contact. D. Lack of the ability to develop peer relationships E. Over 3 years old when symptoms start to develop Ans:- B C D 88) The following condition show characteristic changes on EEG:A. Angelman syndrome B. Infantile syndrome C. Subacute sclerosing panencephalitis D. Petit mal seizures E. Multiple sclerosis Ans:-ABCD 89) Kernicterus is associated with:A. Choreoathetoid cerebral palsy B. Basal staining of the ganglia C. Present in the first few days of life D. Is increased if the mother is given sulfonamide E. Is worse in subsequent pregnancy Ans:-ABCD 90) Concerning cerebral palsy:A. 75% of cases are idiopathic B. Birth weight of less than 1500 g is risk factor C. Mental retardation occur in 20% D. Perinatal asphyxia account for more than 50% E. Prevalence is 1% Ans:- A B C 18

Abdulrahman Bashire

19

Children hospital Benghazi

Perinatal asphyxia account no more than 8 % , Prevalence is 2-4 per 1000 live birth 91) The following are causes of cerebral palsy:A. Hypothyroidism B. Preterm delivary C. Werdning-Hoffmann disease D. Neonatal meningitis E. Congenital CMV infection Ans:- D E Preterm delivary is risk factor for ,but not cause of, cerebral palsy. 92) In children with cerebral palsy: A. All have a degree of learning impairment. B. Birth asphyxia is the most common cause. C. Hand preference before 12 months can be a useful sign. D. The spastic form is the most common clinical type. E. Management should adopt a multidisciplinary approach. Ans:-CDE 93) The following are feature of cerebral palsy diplegia include:A. Toe walking B. Scissoring C. Hip flexion D. External rotation of the leg E. Poor balance Ans:-ABCE internal rotation of the leg 94) In the child with cerebral palsy:A. Spastic diplegia has worse prognosis for intellectual development than other types B. In cases of congenital hemiplegia spasticity is likely to be maximal by the age of 3 months C. Measles vaccine is contraindicated D. Pseudobulbar palsy is likely in cases of congenital hemiplegia E. Sensory loss of cortical type may occur in children with spastic hemiplegia Ans:- E Spastic tetraplegia has worse prognosis for intellectual development the affected limb is likely to be flaccid for the first 2-3 months of life & spaticity only begin to develop from 3 months onwards, becoming maximal at around the age of 12 months pre-existing cerebral palsy isn't contraindication to measles vaccine. Pseudobulbar palsy is condition in which there is bilateral upper motor neuron lesions of the lower cranial nerves ,and thus doesn't occur in hemiplegia. Unilateral cerebral lesion don't causes upper motor lesion of most cranial nerves because of the compensation provided by bilateral cortical innervation. Cortical sensory loss involve two point discrimination,joint sense and stereogenesis. Children with spastic hemiplegia may also have an homonymous hemianopia 95) Kernicterus induced cerebral palsy:A. Is frequently related to damage to basal ganglia B. Is associated with athetoid movement C. Is associated with sever mental retardation D. Can cause defect of upward gaze E. Has an increased incidence of deafness Ans:- A B D E 19

Abdulrahman Bashire 20 Children hospital Benghazi There is high tone hearing loss but intelligence is generally normal,the basal ganglia are often stained with bilirubin & damaged and the dental enamel stained green with poor formed teeth. Children are very thin & lacking SC fat tissue. There is defect of upward gaze ( unlike post anoxic cases of dyskinetic cerebral palsy ) 96) In young children. A. Night terrors occur in the stages of deep sleep. B. Sleep walking occurs during REM sleep. C. Sleep disorders are never treated with drugs. D. Breath holding attacks cause cyanosis. E. Breath holding attacks should be investigated with CT scan. Ans:-AD Eplanation:- Night terrors: child wakes up, frightened and inaccessible, not responding when spoken to, may be difficult to comfort. Period of disturbed behaviour may last up to 15 minutes. Child eventually goes back to sleep. Occur in up to 3% of toddlers. Occur on waking from deep sleep , unlike nightmares that occur in REM or light sleep. Sleepwalking: Occur in a state of deep. May last for a few minutes to half an hour. Behavioural treatments best. Parental reassurance. Alternative of the parents responses to the childrens night time behaviour so as to prevent reinforcement of the problem. Drug treatments not thought to be particularly effective. Breath Holding spells: Common in preschool children. Usually before the age of 2. Precipitated by minor upsets. Follow crying which increases in intensity until breathing stops, usually in expiration, the child goes blue in the face especially around the face. Occasionally, there is a minor epileptic seizure. The child then starts to breath in half a minute or so. Can be used by children to express anger towards parents. Reinforced when parents show concern. 97) Recognized feature of neuro-degenerative disorder in children include:A. Progressive loss\of vision B. Normal intelligence C. Feeding difficulty D. Seizure E. LMNL Ans:-ACD As well as speech and developmental skills which have been acquired in previous years. It associated with UMNL 98) The following statement regarding Rett syndrome are true:A. It is only seen in girls B. Mental handicap is rarely present C. Fits are common D. Macrocephaly is always present E. Respiratory failure is often the cause of death Ans:-ACE 99) Rett syndrome A. Is twice as common in girls as in boys B. Affect children between 4-6 year of age C. Is associated with high ammonia level D. Doesn't effect intelligence E. Is characterized by hand wringing Ans:- E 20

Abdulrahman Bashire 21 Children hospital Benghazi Rett syndrome is neurodegenerative disorder affecting only female,with onset at about 1 year of age, there is loss of purposeful hand movement and communication,skills,social withdrawal, gaite apraxia , stereotypic repetitive hand movements that resemble hand washing, wringing o clapping of hands and acquired microcephaly. The condition plateaus for many years before seizures, spasticity and kyphoscoliosis develop. Metabolic investigation are normal as Rett syndrome is of unknown etiology 100) Rett syndrome:A. Predominantly affects boys B. Manifest in the neonatal period C. Is suggested by autistic mannerisms D. Has pathognomonic EEG abnormality E. Has X linked inheritance Ans:- C 101) The following finding exclude diagnosis of Guillain Barr syndrome :A. Normal CSF B. Ophthalmoplegia C. Autonomic failure D. Sensory level at D10 E. Muscle wasting Ans:-D 102) 5 year old child present with progressive weakness. The following make diagnosis of Guillian Barr syndrome more likely:A. Isolation of C jejuni from the stool. B. Leg pain C. Dilated pupils D. The presence of over 50 cells/ml in the CSF E. Asymmetric weakness of the lower limb Ans:- A B* 103) In the Guillain Barre syndrome in childhood:A. Degree of permanent paralysis is characteristic B. Increased reflexes in lower limb is characteristic C. Paraesthesia are characteristic D. The CSF protein concentration is elevated E. Involvement of the autonomic nervous system is recognized complication Ans:- C D E 104) The following are true of the Guillain Barre syndrome:A. Is associated with symmetrical flaccid paralysis B. Progreesion of the disease for several weeks is common C. Sensory symptoms are absent D. Muscle tenderness exclude the diagnosis E. CSF analysis reveals high cell count but normal CSF biochemistry Ans:- A 105) A 5-yr-old girl is evaluated for severe mental retardation, microcephaly, hand-wringing movements, poor growth and weight gain, and seizures. Genetic analysis indicates a mutation in a transcription factor called MeCP2 that normally functions to silence transcription of numerous genes. This finding indicates that she has which of the following disorders? A. Juvenile Huntington diseas B. Metachromatic leukodystrophy C. Rett syndrome D. Adrenoleukodystrophy

21

Abdulrahman Bashire E. Menkes disease

22

Children hospital Benghazi Ans:- C

Explanation: Rett syndrome is classically described by the features of this case. 106) A mother brings her 6-year-old boy to you for evaluation of repetitive blinking and head turning for the past several months. He is alert during the episodes. The blinking and head turning increase in frequency when he arrives home from school. Of the following, your BEST next step in the evaluation of this patient is to A. ask about any repetitive vocalizations B. ask about exposure to allergens (eg, pet dander, dust) in the home C. ask about stressful changes in the home D. refer him for brain magnetic resonance imaging E. refer him for electroencephalography Preferred Response: A A tic is a sudden, rapid, recurrent, nonrhythmic, stereotyped motor movement or vocalization. Tics may be simple or complex. Examples of simple motor tics are eye blinking, facial grimacing, and shoulder shrugging. Among the common simple vocal tics are throat clearing, sniffing, or chirping. Complex motor tics include hand gestures, twirling while walking, deep knee bends, and sniffing an object. The spontaneous expression of words and phrases is an example of complex vocal tics. Tics often are described as irresistible, although they can be suppressed in situations such as school. Many children have increased frequency of tics after school, when they are at ease at home. Tics may increase in frequency in stressful situations such as before examinations or when children are teased by peers. Tics often decrease in frequency when a child is physically or mentally involved in a task (eg, sports, study). A tic disorder is characterized by motor or vocal tics (not both) that occur daily or nearly daily for at least 4 weeks (transient tic disorder) or for at least 1 year (chronic tic disorder). Onset is before age 18 years, and the tic must not be explained by a medical condition such as stimulant abuse. Tics typically begin between 3 and 8 years of age and are most severe at about age 10 years. Most tics disappear after months. Some may persist into adulthood, but the severity usually diminishes gradually. Children who have Tourette syndrome have both motor and phonic tics, although they need not have both types of tics simultaneously. The tics also occur daily or nearly daily for at least 1 year. During this time, there is no tic-free period longer than 3 months. Onset is before age 18 years, and there should not be another explanation for the disorder. In addition, most children who have Tourette syndrome also have symptoms of attention-deficit/hyperactivity disorder or obsessive-compulsive disorder. The boy described in the vignette has simple motor tics. Differentiating between Tourette syndrome and a tic disorder requires knowledge of whether the boy also is experiencingphonic tics. Many families do not recognize the vocalizations of phonic tics if they are not asked specifically about them. Although this boy does not yet meet the time duration criterion for Tourette syndrome, he and his family should be educated about the natural time course of tic disorders, including Tourette syndrome. Exposure to allergens does not affect the manifestations of a tic disorder. Stressful events may increase the frequency of tics, but they do not cause the disorder. Tics are clinically differentiated from seizures by their nonrhythmic nature and, therefore, electroencephalography is not indicated. Findings on standard neuroimaging are unremarkable in tic disorders, making magnetic resonance imaging unnecessary. 107) Clinical feature of absent corpus collosum include:A. Normal intelligence B. An associated with colobomata of the retina C. Blindness D. Hemihypsarrhythmia on EEG E. Bilateral Erb's palsy Ans:-ABD 22

Abdulrahman Bashire

23

Children hospital Benghazi

108) Contraindication to lumber puncture include: A. Bulging fontanelle in neonates. B. platelet count of 30,000. C. Skin infection over the lumber space D. Marked uncooperativeness of the child E. All children with meningococcal sepsis. Ans:- B C Explanation:- An open fontanelle usually protects against complications of elevated intracranial pressure. However raised intracranial pressure with a closed fontanelle is usually a contraindication to lumber puncture. In an uncooperative child sedation or short anaesthesia may be needed to perform the lumber puncture. Meningococcal sepsis,children can have a lumber puncture provided they are hemodynamically stable and not shocked or in intensive care. Lumber puncture is contraindicted in any child who has an unstable cardiorespiratory status. It should also not be performed in children with uncorrected bleeding diathesis. 109) The following have a much lower concentration in the cerebrospinal fluid (CSF) than in the cerebral capillary blood. A. Potassium B. Sodium C. Protein D. Osmolarity E. Sugar Ans:-C Explanation:- CSF has a very low plasma concentration because large protein molecules cannot cross the blood-brain barrier. The Na+ concentration is roughly the same or slightly more in CSF and the K+ concentration is slightly less in CSF. The CSF sugar content is about two third than that of blood. 110) Characteristic feature of benign intracranial hypertension incude:A. Papilloedema B. UMNL C. V nerve palsy D. Frontal headache E. Large ventricle on CT scan Ans:-AC The headache is generalized headache , V nerve palsy isn't common 111) Regarding the cortico-spinal ( pyramidal) tract:A. 50% originate in the post central gyrus B. 3/4 cross at the medulla C. Lesion only affect proximal region of limbs D. The motor cortex is activated before initiation of the movement E. They are fast conducting tract Ans:-BD The pyramidal tract arise as axons of pyramidal cells situated in the fifth layer of the cerebral cortex,hence 2/3 arise from precentral gyrus & 1/3 arise from postcentral gyrus, the majority of corico-spinal tract are myelinated and are small , relatively slow conductive fibres , 112) Characteristic feature of LMNL include;A. Increase in the tone B. Wasting of the affected muscle 23

Abdulrahman Bashire C. Loss of the abdominal reflexes D. Clonus E. An extensor planter reflex

Children hospital Benghazi

Ans:-B 113) Recognized sign of extrapyramidal tract lesion include:A. Absence of reflexes B. Dysarthria C. Fibrillation D. Cog wheel rigidity E. Hypotonia Ans:-BDE 114) Sensory lesion in the anterolateral column of the spinal cord may characterized by:A. Loss of touch sensation ipsilaterally B. Loss of pain sensation contralaterally C. Vibration sense is abolished ipsilaterally below the lesion D. Exquisite pain on the opposite side E. Astereogenesis Ans:-ABE Vibration sense is abolished ipsilaterally below the lesion occur with posterior column lesion, Exquisite pain on the opposite side occur with thalamic lesion. 115) An extensor plantar reflex commonly occur in:A. Infants B. After generalized convulsion C. Quadriplegic cerebral palsy D. Lower motor neuron lesion E. Comatous chidren Ans:-ABCE 116) The following statement relating to bladder control are correct:A. The sympathetic fibre arise from the 3rd & 4th lumbar segment B. Sympathetic fibres relax the bladder wall C. Pelvic nerves via S2, 3 & 4 relax the sphincter D. Somatic nerves via S2, 3 & 4 relax the external sphincter of the urethra E. Lesion of the anterior sacral nerveroots cause an atonic bladder Ans:-BCE The sympathetic fibre arise from the lumbar segment L1 & L2, Sympathetic fibres relax the bladder wall & contract the sphincter , Pelvic nerves via S2, 3 & 4 relax the sphincter & contract the bladder wall , Somatic nerves via S2, 3 & 4 contract the external sphincter of the urethra 117) The clinical manifestation of lesion of the visual pathway include:A. Optic nerve no light reflex B. Optic chiasma- quadrantic hemianopia C. Optic tract homonymous hemianopia D. Optic radiation ( temporal) - upper quadrant homonymous hemianopia E. Occipital cortex - homonymous hemianopia Ans:-ABCDE 118) Lesion in the parietal lobes are characterized by:A. Apraxia B. Dj vu phenomena C. Perceptual rivalry D. Auditory hallucination E. Apnoea Ans:-ACE Dj vu phenomena temporal lobe , Auditory hallucination temporal lobe 24

Abdulrahman Bashire

25

Children hospital Benghazi

119) Hyper-reflexia is characteristic lesion of :A. Encephalitis B. Duchenne muscular dystrophy C. Cerebral palsy D. Spinal lesion E. Werdnig- Hoffmann disease Ans:-ACD In Spinal lesion in acute stage 120) An upper motor neuron lesion hemiparesis:A. Result in flexion of the wrist and elbow B. May be associated with wasting C. Involve the anterior horn cell D. May be cause of foot drop E. Produce fasciculation Ans;the arm tends to be flexed at the elbow, wrist, and fingers while pronated at the forearm. In UMNL the muscle remain innervated and hypertonic therefore there is no wasting .wasting only occur in LMNL , muscle disease or disuse. 121) Which of the following are inhibitory neurotransmitters in the central nervous system: A. Noradrenaline B. Glutamate. C. Acetylcholine D. Glycine. E. Gama-amino-butyric acid (GABA). Ans:- D E Explanation:Neurotransmitter: Function: Acetylcholine mostly excitatory Bioactive amines: Dopamine excitatory and inhibitory Adrenaline excitatory Noradrenaline excitatory Serotonin excitatory Amino acids: Glutamate excitatory Glycine mostly inhibitory g-Aminobutiric acid (GABA) inhibitor. 122) Noradrenaline A. Is a neurotransmitter at the pre-ganglionic sympathetic nervous system B. Is produced by the adrenal cortex C. May cause a reflex bradycardia when given by infusion D. Causes a rise in metabolic rate E. It acts predominantly via the beta adrenoceptor Ans:- C D Explanation:- Noradrenaline is a catecholamine produced by the adrenal medulla. It also acts as a neurotransmitter in the post-ganglionic sympathetic neurons and parts of the brain. Its actions are mediated mostly through the alpha-adrenoceptor, though it does have some action at the beta-adrenoceptor. It causes vasoconstriction, leading to a rise in blood pressure. It activates the baroreceptor reflex, and hence may cause a reflex bradycardia. It causes an increase in metabolic rate, hence, in the heart, although coronary perfusion may be increased, myocardial oxygen consumption will also be increased. 25

Abdulrahman Bashire 26 Children hospital Benghazi 123) The seventh cranial nerve: A. Has bilateral cortical representation B. Receives taste fibres from the palate. C. When lesioned below the level of the stylomastoid foramen will cause hyperacusis. D. Receives taste fibres from the anterior 2/3rd of the tongue via the greater petrosal nerve. E. Has a sensory supply in the ear canal. Ans:-ABE Explanation:- The difference between the pattern of facial weakness seen in lower and upper motor neurone lesions is explained by the presence of bilateral cortical representation. The seventh nerve receives taste fibres from the palate (via the greater petrosal nerve) and the anterior 2/3rd of the tongue (via the chorda tympani and the lingual branch of the fifth). Herpetic vesicles may be seen in the ear canal in patients with geniculate zoster (Ramsay Hunt syndrome): this is why the sensory supply to this area is important. 124) Complete transaction of the oculomotor nerve results in : A. Ptosis. B. Convergent squint. C. Constriction of the pupil. D. Loss of accommodation and light reflex. E. Diplopia. Ans:-ADE Explanation:- The nucleus of the third nerve lies ventral to the aqueduct in the midbrain. Efferent fibres to four external ocular muscles (superior, inferior and medial recti, and inferior oblique), levator palpebrae superioris and sphincter pupillae (parasympathetic) enter the orbit through the superior orbital fissure \9from kumar and clark). A complete oculomotor palsy results in the affected eye being displaced down and out with a dilated pupil. 125) With regard to the visual pathways: A. A lesion of the optic chiasma will cause a homonymous hemianopia B. A lesion of the left occipital lobe will result in a right homonymous hemianopia C. The pupillary light reflex is preserved in lesions of the optic nerve D. Lesions of the optic nerve cause an enlargement of the blind spot E. A pituitary adenoma may cause a bitemporal hemianopia Ans:- B D E Explanation:- The visual pathway runs from the eye to the occipital cortex. The light reflex gives an indication of optic nerve function and will, therefore, be abnormal in lesions of the optic nerve; in addition there will be enlargement of the blind spot. Chiasmal lesions (e.g. due to compression by pituitary tumors) result in bitemporal hemianopia. A lesion of the optic tract/radiation or occipital cortex will result in a contralateral homonymous hemianopia. If incomplete, a lesion of the optic radiation with result in a contralateral upper quandrantinopia if in the temporal region, or lower quadrantinopia if in the parietal region. 126) Pupils: A. Are usually small reactive in medullary lesions B. Are usually pinpoint in anticholinergic poisoning. C. Typically dilate unilaterally in III nerve lesions. D. Large pupils are seen with amphetamine and tricyclic antidepressant ingestion. E. The parasympathetic Edinger-Westphal nucleus is involved in light and accommodation reflexes. Ans:- A C D E Explanation:- Fixed dilated pupils are seen with anticholinergic, hypothermia, sever hypoxia, barbiturates (late sign), during and after seizures. 26

Abdulrahman Bashire

27

Children hospital Benghazi

127) The following statements are correct regarding Horners syndrome: A. It may be caused by injury to the superior cervical sympathetic ganglion. B. Ptosis occurs because of loss of innervation to the smooth muscle portion of the levator palpebrae superioris. C. Pupillary dilatation occurs because of loss of innervation to the iris sphincter muscle. D. May be associated with ophthalmoplegia in the territory of the IIIrd cranial nerve. E. May be associated with loss of sweating on the ipsilateral arm. Ans:- A B E Explanation:Horner's syndrome may be caused by any injury to the sympathetic chain in the neck. It is associated with ipsilateral ptosis (paralysis of levator palpebrae superioris), meiosis (paralysis of dilator pupillae muscle) and facial and upper limb anhidrosis. The somatic component of the IIIrd nerve is unaffected and thus ophthalmoplegia is not a feature 128) Horners syndrome invariably produces :A. Ipsilateral ptosis B. Contralateral mydriasis C. Loss of sweating over the ipsilateral face D. Heterochromia E. Exophthalmos Ans:- A Explanation:- Horners syndrome consists of ptosis, miosis and anhydrosis and is caused by interruption of the sympathetic supply to the eye. The pathway starts in the ipsilateral hypothalamus and has three neurones with synapses in the lateral grey of the spinal cord at T1 and superior cervical sympathetic ganglion. Causes depend on the level of the lesion:a) Brainstem: tumour, demyelination, infarction b) Cervical cord: syringomyelia, tumour, demyelination c) T1: apical carcinoma of lung (Pancoast syndrome), cervical ribs, trauma d) Cervical sympathetic chain: thyroid carcinoma, carotid body tumour, neck surgery e) Internal carotid: dissection, jugular foramen syndromes All the features of Horners are ipsilateral. Loss of sweating may not occur if the lesion is distal to the superior sympathetic ganglion. Heterochromia refers to different pigmentation of the iris compared to the normal eye and is a feature of congenital Horners. The affected eyeball often seems sunken (enophthalmos) owing to slight elevation of the lower eyelid. 129) A patient develops low back pain which is progressive. He is found to a spinal schwannoma. It is resected, but division of the posterior spinal root occurs. Which of the following is likely to result from this? A. Loss of motor function B. Loss of tendon reflex C. Loss of pain, temperature and preservation of light touch D. Spinothalamic tract degeneration above the section level E. Absence of sweating in the limb Ans:- B D Eplanation:- Division of dorsal/posterior root usually causes loss of all sensory modalities in a dermatomal distribution, which makes it different from peripheral neuropathy which is not dermatomal, and usually not entirely sensory or motor. Posterior division causes spinothalamic tract degeneration. There is no effect on the sympathetic supply, as

27

Abdulrahman Bashire 28 Children hospital Benghazi 130) In the examination of the cranial nerves:A. An ipsilateral cranial nerve lesion with contralateral hemiplegia suggest lesion in the brain stem B. Unilateral UMNL of the X cranial nerve are not clinically detectable C. Bitemporal hemianopia suggest lesion in the optic radiation D. Lesion of the V cranial nerve cause lateral rectus palsy E. Loss of tast to the anterior 2 / 3 of the tongue can occur in lesion of the V cranial nerve Ans:-ABE 131) Lesion of the right V cranial nerve:A. Can cause diplopia when looking to left B. Can cause convergent strabismus C. Neither eye will be able to cross the midline D. Causes diplopia with the images separating further on gaze to the right E. May be due to raised ICP Ans:- B D E Right abducens palsy will paralyse abduction of the right eye, thus its effect will be most marked on looking to the right unopposed action of the right medial rectusmay result in convergent strabismus 132) The facial nerve:A. Subserve no cutaneous sensory function B. Give motor supply to the inner ear C. Control the movement of the anterior two third of the tongue D. Carries parasympathetic fibres to the salivary gland E. Innervates platysma Ans:- D E Although predomintly motor ,the facial nerve has clinically important cutaneous sensory supply to the skin of the external auditory canal ( from the vagus) vesicle in this area combined with facial palsy are diagnostic of the Ramsay Hunt syndrome ( geniculate herpes zoster) Give motor supply to the middle ear, nerve to the stapedius the facial nerve conveys sensory fibres ( taste) via chorda tympani- from the anterior two third of the tongue. The motor supply to the whole of the tongue is via the X cranial nerve 133) The pupillary light reflex depend upon the integrity of :A. The optic nerve B. The lateral geniculate nuclei C. The occipital cortex D. The medial longitudinal fasciculus E. The ( oculomotor ) cranial nerve Ans:-A B E v The sensory component of the papillary light reflex is conveyed from the retina via the optic nerve to the chiasm,here the nerve decussates and conveys impulses to both optic tract and both lateral geniculate nuclei, here some fibres are relayed to both Edinger-Westphal nuclei( the nerve nucleus) and complete the motor component of the light reflex. v The patient can be cortically blind due to occipital cortex destruction but will still have intact papillary reflexes v The medial longitudinal fasciculus is involved in contributing conjugate eye movement. 134) A 3 years old child presents with progressive gait difficulty, absent tendon reflexes in the lower limbs, recurrent falls, pes cavus, and abnormal nerve conduction study. The differential diagnosis should include: A. Ataxia telengietasia. B. Meachromatic leukodystrophy C. Guillain-Barre syndrome D. Hereditary motor sensory neuropathy. 28

Abdulrahman Bashire E. Cerebral palsy.

29

Children hospital Benghazi

Ans:- B D Explanation:- The hereditary motor sensory neuropathies (HMSN) are a group of genetic disorders with incidence of 10 per 100,000 populations. The commonest variant is HMSN type I: Charcot-Marie-Tooth disease. It is autosomal dominant, and in many cases the abnormal gene is on chromosome 17. The initial symptoms are either foot deformities or gait disturbances beginning in the second decade or, sometimes, in early infancy. Pes cavus is typical and hammertoes may be present. Initial examination will show atrophy of peroneal muscles, pes cavus and reduced reflexes of tendon Achilles. The disease is progressive and leads to progressive muscle weakness, which may affect the proximal as well as distal muscles. Foot drop and cramps. In adult, preipheral nerves are enlarged due to repeated episodes of demyelination and remyelination. Nerve conduction study will be diagnostic as well as muscle biopsy in cases with normal nerve conduction study. There is no specific treatment but supportive care with proper foot care. Other types of HMSN include HSMSN type II, HSMSN type III, (Dejerine-Scoots disease), HSMSN type IV (Refsums Disease). Metachromatic leukodystrophy can present in a similar manner. 135) Clinical examination of cerebellar function: A. Head tilt usually ipsilateral to the involved hemisphere. B. Tremor is seen at rest. C. Hypotonia may be present. D. Rhomberg test is positive. E. Acute ataxia is seen with varicella, mumps, typhoid and pertussis. Ans:- A C E Explanation:- Cerebral neoplasms may also result in head tilt by tonsillar herniation with raised pressure. Intention tremor is seen in cerebellar dysfunction. Rhomberg test is positive (unsteady with eyes closed ) in posterior column dysfunction and sever peripheral neuropathy. In cerebellar dysfunction, the child is unsteady with eyes open and closed. 136) The following condition cause nystagmus A. Basilar artery migraine B. Friedreich ataxia C. Phenytoin toxicity D. Congenital cataract E. Guillain Barr syndrome Ans:-ABCD 137) The following condition typically cause cerebellar ataxia in childhood: A. Medulloblastoma B. Meningioma of the falex cerebri C. Polio D. Varicella encephalitis E. Dystonia musculorum deformans Ans:-AD 138) Recognized cases of acute ataxia include:A. Occult neuroblastoma B. Paracetamol overdose C. Varicella infection D. Maple syrup urine disease E. Refsum 's disease Ans:- A C D Refsum 's disease cause chronic progressive ataxia 139) Acute ataxia may be caused by:A. Alcohol ingestion 29

Abdulrahman Bashire B. Hypoglycemia C. Cerebellar astrocytoma D. Neuroblastoma E. Phenytoin

30

Children hospital Benghazi

Ans:-ABCDE 140) Ataxia Telangiectasia:A. Is an autosomal dominant condition B. Usually present with cerebellar ataxia C. Typically has classic skin lesions clearly visible by 3 years D. Is associated with fitting in the majority of cases E. Doesn't affect intelligence Ans:- B 141) Ataxia Telangiectasia is associated with:A. Nasal sinusitis & middle ear disease B. Tonsillar hypertrophy C. Defective immunoglobulin synthesis D. Development of leukemia E. Glucose intolerance Ans:-ACDE The is reduced or absent tonsillar tissue & frequently absent thymus, 142) Features of Friedreich ataxia include:A. Prominent deep reflex B. Pes cavus C. Extensor plantar response D. Steatorrhea E. Dysartheria Ans:- B C E The Friedreich ataxia is cause of progressive ataxia 143) Concerning Fredrich's ataxia:A. The gene locus is known B. Plantar are down going C. Third of patient develop malignancy D. Onset is usually by the age of 5 E. Pes cavus is recognized finding Ans:- A E The onset of ataxia usually occurs before age 10 yr. The ataxia is slowly progressive and involves the lower extremities to a greater degree than the upper extremities. The Romberg test result is positive; the deep tendon reflexes are absent (particularly the Achilles), and the plantar response is extensor. Patients develop a characteristic explosive, dysarthric speech, and nystagmus is present in most children. Although patients may appear apathetic, their intelligence is preserved. They may have significant weakness of the distal musculature of the hands and feet. Typically noted is a marked loss of vibration and position sense caused by degeneration of the posterior columns and indistinct sensory changes in the distal extremities. Friedreich ataxia is also characterized by skeletal abnormalities, including high-arched feet (pes cavus) and hammertoes, as well as progressive kyphoscoliosis. Results of electrophysiologic studies including visual, auditory brainstem, and somatosensory-evoked potentials are often abnormal. Hypertrophic cardiomyopathy with progression to intractable congestive heart failure is the cause of death for most patients. Antioxidant therapy with coenzyme Q10 and vitamin E has been reported to slow progression in some patients. 144) Down beat nystagmus is feature of:A. Type chiari malformation B. Dysgerminoma C. Phenytoin toxicity 30

Abdulrahman Bashire 31 D. Aquaductal stenosis E. Lesion of medial longitudinal fasciculus

Children hospital Benghazi

Ans:-ACD Down beat nystigmus\is accentuated by vertical gaze. Localizes disease to the medulla of the brain stem( although brain-stem disorder more commonly produce horizontal nystigmus) , lesion to the medial longitudinal fasciculus cause impaired convergence and lateral conjugate gaze, if the lesion is in the lower part of the tract then V cranial nerve will also be involved & there will be defective abduction causing horizontal nystigmus. 145) Regarding the meninges: A. The dura is adherent to the bone B. Subdural hematomas form a lentiform shape C. Arachnoid folds project inwards to form the falx cerebri and falx cerebelli D. Pia stimulation during surgery is very stimulating E. Pia is responsible for production of CSF Ans:- A E Explanation:* The dura's rich arterial system supplies the bone and dura. * Extradural hematomas form a lentiform shape; subdurals follow the form of the cerebral hemispheres. * The dura project inwards to form the falx. * Dural stimulation is very stimulating, and depth of anesthesia may need to be altered in response. 146) Cerebral blood flow: A. Is increased in an acutely compensated metabolic acidosis. B. Is decreased at 72 hours in a normal person who is made to hyperventilate for 72 hours. C. Shows a right shift in the autoregulation curve in patients with chronic hypertension. D. Shows a linear increase with PaO2. E. Is unaffected by temperature. Ans:- C The CBF is affected by PaO2, PaCO2, MAP, ICP, temp, cerebral metabolism, neurogenic factors and drugs. 1. A compensated metabolic acidosis is achieved via hyperventilation, which will decrease the PaCO2. As it is acute there will be no change in the CSF pH. 2. Studies have shown that although there is the initial cerebral vasoconstriction with the low PaCO2, this gradually normalizes within 72 hours despite the low PaCO2. 4. The CBF shows an exponential increase in the CBF at PaO2 levels of below 5.5 Kpa but is fairly constant at levels above that. 5. Every decrease in body temperature of 1 degree celcius will decrease the cerebral metabolism by 7%. 147) Strokes in childhood are associated with :A. Nephrotic syndrome B. Activated protein C resistence C. Systemic lupus erythematosis D. Haemoglobin SS E. Moya moya disease Ans:-ABCDE Explanation:- Strokes are rare in childhood. Due to arterial occlusion, venous thrombosis. Causes: -sickle cell disease -arteriovenous malformations and vascular dysplasia eg moya moya disease -vasculitides eg SLE, PAN -coagulopathies eg AT III, protein C and S deficiency, activated protein C resistence. -dehydration -trauma -metabolic disorders eg MELAS, homocysteinuria 31

Abdulrahman Bashire

32

Children hospital Benghazi

148) Strokes in childhood are associated with :A. Nephrotic syndrome B. Activated protein C resistence C. Systemic lupus erythematosis D. Haemoglobin SS E. Moya moya disease Ans:- A B C D E Explanation:- Strokes are rare in childhood. Due to arterial occlusion, venous thrombosis. Causes: -sickle cell disease -arteriovenous malformations and vascular dysplasia eg moya moya disease -vasculitides eg SLE, PAN -coagulopathies eg AT III, protein C and S deficiency, activated protein C resistence. -dehydration -trauma -metabolic disorders eg MELAS, homocysteinuria 149) Moyamoya disease:A. Has wide range of symptoms on presentation B. Tend to resolve spontaneously C. May result in an acute hemiplegia D. Is associated with neurofibromatosis E. Can be safely & non-invasively investigated by hyperventilation during EEG monitoring Ans:- A C D 150) Fasciculations may be seen in: A. Motor neurone disease. B. Cervical spondylosis. C. Hypomagnesaemia. D. Hypocalcaemia E. Thyrotoxicosis. Ans:- A B C D E Explanation:- Fasciculations are caused by spontaneous contractions of individual motor units, and may occur in any cause of subacute partial denervation or tetany. Metabolic causes include thyrotoxicosis and hyponatraemia, hypocalcaemia and hypomagnesaemia. Drugs associated with fasciculations include colfibrate,salbutamol (especially iv therapy), and lithium toxicity. 151) The following are recognized causes of floppy infant syndrome:A. Dystonia musculorum deformans B. Maternal myasthenia gravis C. Limb girdle muscular dystrophy D. Maternal benzodiazepine therapy E. Werdnig-Hoffman disease Ans:- B D E 152) Recognized cause of intracranial calcification:A. Histocytosis X B. Craniopharyngioma C. Congenital toxoplasmosis D. Marble bone disease E. Hurler syndrome Ans:- B C 32

Abdulrahman Bashire 33 Children hospital Benghazi 153) Concerning brain growth development and function in infancy:A. Maximal neuronal proliferation occur in the third trimester of pregnancy B. The newborn brain is less susceptible than adult brain to the effect of hypoxia C. Head circumference increase in linear fashion over the first 2 year of life D. The main changes in the brain over the age from 0 to 2 years are myelination and glial proliferation E. Convulsion can present in the newborn as apnoeic attack Ans:- B D E The maximum number of brain cells is attained by 30 weeks of gestation and all subsequent changes consist of increased differentiation ,myelination and glial proliferation. The head circumference increase very rapidly over the first 6 months then gradually slow over the next 18 months 154) Which of the following are common symptoms associated with raised intracranial pressure in children? A. Morning headache B. Abdominal pain C. Double vision D. Ataxia E. Seizure Ans:- A C Eplanation:- Benign intracranial hypertension is characterised by increased intracranial pressure in the absence of a mass lesion or hydrocephalus. It is idiopathic in 50%. Secondary causes include: * Drugs (corticosteroids, tetracycline, vitamin A and oral contraceptives); * SLE; * Head trauma; * Otitis media; * Sinusitis; * Adrenal insufficiency; * Hyperthyroidism; * Hypoparathyroidism; * Hyperadrenalism; * Leukaemia; * Polycythemia; * Guillain-Barr syndrome; * Iron deficiency anaemia. Headache is the commonest feature; others include diplopia, caused by 6th nerve palsy (abducens). Less commonly, some may have nausea, vomiting, paraesthaesia, neck stiffness, and ataxia. Papilloedema is a common early finding. If not treated, the papilloedema will progress and cause optic nerve atrophy. Cranial CT or MRI is essential with ophthalmology review. Lumbar puncture will help to relieve headache and preserve the vision. If the pressure is high, it should be reduced by 40% - the normal pressure lies between 12-16 mmHg. Acetazolamide can be started if patient cannot tolerate second or third lumbar puncture when symptoms recur. In spite of all these, if a child is still symptomatic with progressive loss of vision, dexamethasone can be given for two weeks or/and consider optic nerve fenestration or lumboperitoneal shunt. 155) A 7-year-old boy develops mental status changes 2 days after surgery to remove a craniopharyngioma. Two days later, he develops hyponatremia (serum sodium of 124 mEq/L [124 mmol/L]), with an elevated serum creatinine (1.2 mg/dL [106.1 mcmol/L]). His urine output increases from 2.2 mL/kg per hour to 4.8 mL/kg per hour, and his weight decreases from 18.8 kg to 18.0 kg. His serum osmolality is 620 mOsm/L, and his urine osmolality is 345 mOsm/L. His vital signs include a temperature of 98.9F (37.2C), heart rate of 128 beats/min, and blood 33

Abdulrahman Bashire 34 Children hospital Benghazi pressure of 80/50 mm Hg.Of the following, the MOST likely cause of the abnormalities described for this child is:A. adrenal insufficiency B. cerebral salt wasting C. diabetes insipidus D. hemolytic-uremic syndrome E. syndrome of inappropriate secretion of antidiuretic hormone Preferred Response: B The child described in the vignette has features that are typical for volume depletion, as occurs in cerebral salt wasting. This disorder may be associated with space-occupying lesions such as intracranial tumors. Adrenal insufficiency results in hyponatremia, but is not a sequela of an intracranial mass. In diabetes insipidus, the massive losses of water result in hypernatremia, not hyponatremia. In hemolytic-uremic syndrome, the serum creatinine is elevated, but the urine output is decreasing, not increasing. Because antidiuretic hormone stimulates reabsorption of water in the kidney, the clinical signs of the syndrome of inappropriate secretion of diuretic hormone (SIADH) are attributable to retention of excess amounts of water. Thus, a patient who has SIADH usually exhibits signs of volume overload, including high blood pressure, low or normal heart rate, increased weight, and reduced urine output. Hyponatremia occurs because the excess water dilutes the serum sodium concentration. The blood urea nitrogen (BUN) value may be very low, and the serum creatinine measurement generally is normal. The urine sodium concentration is either normal or elevated, despite the low serum sodium concentration. Because the kidney is excreting less water than under normal conditions, the urine specific gravity is very high (generally greater than 1.020), and the urine osmolality is elevated. Patients who have SIADH have a higher urine versus serum osmolality (generally 1.5 to 2 times the serum osmolality). Other biochemical features of SIADH include elevated serum aldosterone (to retain sodium in the kidney) and low plasma renin activity. The serum uric acid concentration usually is decreased (dilutional effect), and the serum atrial natriuretic peptide concentration is increased. The latter is an attempt by the heart to correct the volume overload by promoting salt excretion. 156) Concerning brain tumours in children: A. A sixth cranial nerve palsy can be the presenting feature. B. The majority are metastatic from other tumors. C. They are the most common solid organ tumor. D. Brain stem glioma usually present with personality change. E. Medulloblastomas can metastasize to the spine. Ans:-ACE 157) The following criteria must exist before a diagnosis of brain death can be made: The following criteria must exist before a diagnosis of brain death can be made: A. Fixed, dilated pupils unresponsive to light. B. Absent gag reflex with pharyngeal stimulation C. Neuromuscular drugs should have been stopped for at least twelve hours. D. No eye movement when 20 mls cold water is perfused into the external auditory meatus. E. No motor response from painful stimuli. Ans:- ADE Explanation:- An absent gag reflex must be assessed with bronchial stimulation via a tracheal catheter. Neuromuscular drugs should have been stopped for at least 24 hours before assesment can be made. 158) Attention deficit hyperactivity (ADHD):A. Has an incidence in the childhood populationof between 1 and 5 per 1000 B. Remit spontaneously in aadolescent C. Has recognized association with specific learning difficulty D. Has higher incidence in social classes & 34

Abdulrahman Bashire 35 E. Include impulsivity as ones of its 159) Autism :A. B. C. D. E. Can be treated successfully with psychotherapy Is sometimes related to poor parenting Is related to Fragile X syndrome Is related to Sandifer Syndrome Is related to Rett syndrome

Children hospital Benghazi cardinal features Ans:-CE

Ans;- C E Eplanation:- Features impairments in socialization, communication and imagination. More common in boys than girls. Causes: genetic (rate of autism in siblings increased, autism is a feature of fragile X), infective aetiology, immunological theories. NOT related to parenting methods, child abuse or neglect. Usually diagnosed around 2-3 years of age as that is when speech delay becomes readily apparent. Social interaction: autistic aloneness. Inability to recognise or appreciate the feelings of others and respond to social cues. Unable to modify their behaviour on the basis of these social cues. Parents often report child not wanting to be cuddled as an infant. Late development of social smile. Gaze avoidance. Poor development of joint attention, protodeclarative pointing. Communication: verbal and non-verbal language affected. Poor speech development. Poor development of facial expression, use of gestures. abnormal use of speech- echolalia. Imagination: restricted range of interests. Stereotypies eg head banging, rocking, hand-flapping. Inflexible routines. Inability to adapt to change. ritualistic behaviour. Poor symbolic or imaginative play. Aspergers syndrome: actually modern view suggests that Aspergers describes the milder autistic children ie an autistic spectrum of disorders exists. Aspergers describes those children who have impairments in the core features without learning difficulties. They tend to be diagnosed later because they are able to cope and adapt better than severe autistics. Retts syndrome: occurs in girls. Initially normal development with regression and loss of skills before 2 years of age, especially language and manual dexterity skills. Autistic features. Treatment of developmental disorders: symptomatic treatment as causes poorly defined. Educational and behavioural methods are mainstay of treatment. 160) Autism:A. Is condition in which it is desirable to keep affected children in mainstream schools B. Can occur in children who have shown normal development for the first four years of life C. Characteristically affects linguistic communication while sparing non- verbal communication D. Characteristically occur in children of above average IQ E. Is thought to be the result of extreme deprivation Ans:-all false 161) In the treatment of attention deficit hyperactivity disorder :A. Behavioural therapy is never useful B. Pharmacological intervention includes the use of stimulant drugs such as amphetamines C. Methylphenidate (Ritalin) should not be given later than 4pm D. Parenting techniques can be taught E. Poor appetite is a common side affect of methylphenidate (Ritalin) Ans:- BCDE Explanation:- Treatment of attention deficit hyperactivity disorder: stimulant medication- methylphenidate (Ritalin), dexamphetamine. Not to be taken after 4pm. Side effects include difficulty sleeping, loss of appetite, abdominal pain, headache, thrombocytopenia. Behavioural modification, parenting techniques. Outcome: symptoms tend to lessen as child gets older. 162) Concerning ADHD (attention deficit hyperactivity disorder): A. Impulsive behaviour is a feature. 35

Abdulrahman Bashire 36 Children hospital Benghazi B. Is associated with a higher rate of expulsion from school. C. Is associated with a higher rate of teenage pregnancy. D. Is associated with a higher rate of tic disorders. E. May respond to treatment with stimulants such as amphetamines. Ans:-ABCDE 163) Regarding anorexia nervosa: A. Is never fatal. B. Occurs exclusively in females. C. Amenorrhoea can be a feature. D. Growth hormone is typically elevated. E. Distorted body image is common. Ans:-CDE 164) The following may be diagnostic significance in the assessment of hypotonic neonate:A. Serum creatine kinase B. Teenage parents with strong aura of social problems around them C. Relative sparing of muscles of facial expression D. Karyotype analysis E. Poor feeding Ans:-ABCDE Maternal use in the last trimester of either narcotic drugs or benzodiazines could be responsible for the infant being floppy Relative sparing of muscles of facial expression is characteristic of Werdnig-Hoffmann disease Prader-Willi syndrome is an important condition presenting with neonatal hypotonia, the diagnosis can be confirmed by demonstrating the 15q deletion syndrome 165) A lesion in the spinal cord at C8 would give: A. Loss of sensation over the lateral aspect of the hand. B. Loss of bronchioradialis reflex. C. Weakness of abduction of shoulder D. Weakness of flexion of finfers. E. Adduction of the fingers. Ans:- D E Explanation: C8 supplies sensation to the medial hand, the lateral aspect is supplied by C6. Bronchioradialis is supplied by the radial nerve from C5 and C6. Abduction of the arm is mediated by C5 and C6 through supraspinatus and deltoid. Finger flexion is supplied by the median and ulnar nerves, both of which involve C8 root supply. The small muscle of the hand are supplied by the ulnar nerve (C8,T1) 166) A 15 years old boy from Ghana presents with diplopia and ptosis. His symptoms are much worse at the end of the day. Which of the following statements are true of myasthenia gravis: A. Is due to a disorder of peripheral nerve. B. Acetylcholine antibodies are always present. C. May respond to treatment with anticholinergic drugs. D. The electromyogram may be abnormal. E. May be due to transplacental passage of IgG antibodies. Ans:- D E Explanation:- myasthenia gravis is a disorder of the neuromuscular junction, most commonly related to antibodies to the acetylcholine receptor. However, these antibodies may be absent, particularly in childhood or in congenital forms. The hallmarks are weakness and fatigability of muscles. Treatment is with acetylcholinesterase inhibitors. Mothers with myasthenia may transfer IgG antibodies across the placenta to their infants who them have transient neonatal myasthenia. The EMG shows a decremental response to repetitive stimulation 36

Abdulrahman Bashire

37

Children hospital Benghazi

167) You are following a 1-year-old girl who was admitted to the hospital with bronchiolitis. She had cardiac surgery to repair tetralogy of Fallot 6 months ago and had an uneventful recovery. Growth parameters are at the 25th percentile, and neurologic examination findings are normal. Findings on genetic evaluation are normal. A resident physician asks you about the childs predicted cognitive abilities and school performance.Of the following, your BEST response is that:A. cognitive function cannot be predicted at this early age B. her academic performance depends primarily on the amount of early intervention services she receives C. magnetic resonance imaging is the best predictor of future cognitive function D. she likely will have difficulty with attention and learning E. she will have normal cognitive abilities Preferred Response: D Children who have cyanotic congenital heart lesions are receiving surgery at earlier ages, which has improved survival, but they are at risk for developmental delay and poor school performance related to their condition. Although it is not possible to predict a specific outcome for the child described in the vignette, most children who have congenital cyanotic heart disease have some degree of cognitive impairment. All of the factors that affect cognitive development in these children are not well understood, but outcome is related to the degree of preoperative hypoxia, intraoperative management (cardiac bypass time), and postoperative complications. Children who have cyanotic heart lesions and abnormal findings on neurologic examination have poorer school performance than do children who have similar lesions and normal neurologic examination findings. However, children who have cyanotic heart lesions and normal neurologic examination findings have poorer school performance than do children who have heart lesions that do not require surgical intervention. Children who have acyanotic heart disease have a better academic prognosis than do children who have cyanotic heart disease. The child described in the vignette probably will have difficulty with attention and learning in the classroom. Magnetic resonance imaging occasionally may detect additional abnormalities, but it is not the best predictor of future cognitive function. The child in the vignette should be monitored closely for developmental progress and referred for early intervention services if any concerns develop. If she qualifies for services, such support is important to her development. However, there is no direct correlation between hours of services received and academic outcome.

QUESTION NUMBER 5112 previously healthy 14-year-old girl had been evaluated for hoarseness. A right vocal fold impairment was noted on physical examination. Magnetic resonance imaging (MRI) detected multiple intracranial lesions that were consistent with neurofibromoatosis type 2. Whiich of the following nerves are likely to be involved in this girl? a) superior vestibular branch of the 8th cranial nerve b) inferior vestibular branch of the 8th cranial nerve c) superficial petrosal nerve d) auriculotemporal branch of the facial nerve e) Posterior auricular branch of the facial nerve Correct answer: A EXPLANATION 37

Abdulrahman Bashire 38 Children hospital Benghazi Neurofibromatosis type 2 is an often devastating autosomal dominant disorder which, until relatively recently, was confused with its more common namesake neurofibromatosis type 1. Subjects who inherit a mutated allele of the NF2 gene inevitably develop schwannomas, affecting particularly the superior vestibular branch of the 8th cranial nerve, usually bilaterally QUESTION NUMBER 1057 A one year old male child has multiple symmetrical joint contractures of the upper and lower limbs, most marked in the lower limbs, and distributed distally rather than proximally. His subcutaneous tissues are thick and feel doughy. He is also noted to have clubbing. A likely diagnosis is a) amyoplasia b) Ehler-Danlos syndrome c) congenital indifference to pain d) Hurler's syndrome e) Marfan syndrome Correct answer: A EXPLANATION The cause of amyoplasia is unknown, but children with this disorder have a decreased number of anterior horn cells in the spinal cord, suggesting a neuropathic cause. Other studies have shown that the disorder may be myopathic in origin. In the latter instance, diminution of in utero movement may be the final common pathway leading to contractures. Every child presenting with multiple joint contractures should have a complete musculoskeletal evaluation and genetics consultation. The distribution of involvement of multiple joint contractures is variable. The classic presentation involves the upper and lower extremities. The lower extremities are typically more involved than the upper extremities. Involvement of all four extremities is quadrimelic involvement. It is also possible that only the lower extremities or upper extremities (bimelic) may be involved. It is unusual to see only one extremity or a portion of one extremity (monomelic) involved. QUESTION NUMBER 4813 A young man has an inherited multi-system disorder characterized by angiomas in the retina , adrenal glands and the spinal cord. This disorder has been associated with a) mutation of TSC1 and TSC2 genes b) mutation of DPC4 gene c) Mutation of p21 gene d) Mutation of Rb gene e) Mutation of VHL gene Correct answer: E EXPLANATION Von Hippel-Lindau syndrome is an inherited multi-system disorder characterized by abnormal growth of blood vessels. While blood vessels normally grow like trees, in people with VHL little knots of blood capillaries sometimes occur. These knots are called angiomas or hemangioblastomas. Growths may develop in the retina, certain areas of the brain, the spinal cord, the adrenal glands and other parts of the body. The gene for Von-Hippel Lindau disease (VHL) is found on chromosome 3, and is inherited in a dominant fashion. If one parent has a dominant gene, each child has a 50-50 chance of inheriting that gene. The VHL gene is a tumor suppressor gene. This means that its role in a normal cell is to stop uncontrolled growth and proliferation. If the gene is lost or mutated, then its inhibitory effect on cell growth is lost or diminished, which, in combination with defects in other regulatory proteins, can lead to cancerous growth. LIke the Rb tumor suppressor gene, VHL seems to act as a 'gatekeeper' to the multistep process of tumorigenesis. QUESTION NUMBER 933 Neisseria meningitides is a gram negative organism which causes meningitis in young adults. The bacteria are most likely to gain entry into the meninges via: a) the blood stream 38

Abdulrahman Bashire 39 b) the lymphatic drainage of the meninges c) the perineural sheath of the olfactory nerve d) the neural tracts of the spinal cord e) the optic nerve and tract

Children hospital Benghazi

Correct answer: C EXPLANATION Neisseria meningitides or Meningococci are strict human parasites which often cause meningitis in young adults. The bacteria gain entry into the human body through the nasopharynx where they cause a mild infection with rhinitis and pharyngitis. Here the organisms multiply and then gain access to the meninges. The organism enters the meninges along the perineural sheath of the olfactory nerve, through the cribriform plate of the ethmoid.Spread through the blood is very unlikely because the Meningococci do not cause a septicemia following the nasal infection which is essential in case the organism enters the meninges through the blood. Meninges and nasopharynx do not share their lymphatic drainage so entry through lymphatics is not possible.Meningococci do not infect the peripheral nervous system so entry through the spinal cord is not possible. QUESTION NUMBER 1093 An 16 year old girl complains of constant headache disturbing normal sleep patterns and made worse by sneezing or coughing. She also complains of blurred vision and photophobia. On examination she is found to have problems with balance and spatial awarenessand a loss of short-term memory. Fundoscopy reveals papilloedema. A CT scan is normal. The CSF pressure is found to be raised on lumbar puncture. A likely diagnosis is a) medulloblastoma b) pseudotumour cerebri c) encephalitis d) aneurysmally dilated great vein of Galen e) thrombosis of a major dural sinus Correct answer: B EXPLANATION Adolescents with pseudotumor cerebri (benign intracranial hypertension) present with an insidious onset. Patients have increased headache when coughing, straining with bowel movements, or sneezing. Papilledema should be present, and an LP with an opening pressure above 20 cm water confirms the diagnosis The symptoms reported are those usually associated with raised intracranial pressure (ICP); these will commonly include headache, visual disturbances, photophobia, vomiting, problems with balance and spatial awareness, disorientation, loss of short-term memory (sometimes long-term memory loss), pins and needles or loss o sensation in hands. It is important to exclude cerebral tumour as a cause of the symptoms People with raised ICP may find it difficult to cope with previously learnt everyday tasks, eg handling money, using the telephone. They may be unable to find their way around a previously familiar town: traffic is confusing, they can be unaware of kerb height (sometimes afraid to step into space); crossing the road can be a nightmare. Diagnosis of BIH is by scan and measurement of the CSF pressure. On CT scan, the ventricles (chamber in the brain) will usually appear normal or small. The CSF pressure should always be measured despite the norma scan results. It will be found to be raised on lumbar puncture. On examining the eyes, papilloedema (swelling of the optical disc) may be present. This in itself is often an indication for surgical intervention. QUESTION NUMBER 1078 a six week old male infant is noted to be tacchypneic and distressed. A CXR demonstrates cardiomegaly with right sided predominance, as well as superior mediastinal widening. A possible cause includes a) Moya Moya disease 39

Abdulrahman Bashire 40 b) pseudotumour cerebri c) encephalitis d) aneurysmally dilated great vein of Galen e) thrombosis of a major dural sinus

Children hospital Benghazi

Correct answer: D EXPLANATION An aneurysmally dilated great vein of Galen causes high output congestive heart failure in neonates whilst in older children it obstructs the Aqueduct of Sylvius, producing non communicating hydrocephalus. This malformation is a congenital arteriovenous malformation with blood shunting from cerebral arteries into an aneurysmally dilated great vein of Galen and the straight sinus. The major feeding vessels are the posterior cerebral artery and its branches the posterior choroidal and posterior perforating artery Chest film shows cardiomegaly with right sided predominance. The superior mediastinum is widened due to dilation of the brachiocephalic arteries and veins. Pulmonary vascularity is frequently normal because of the pulmonary arterial hypertension of the newborn. On angiography rapid arteriovenous shunting into a dilated saclike vein of Galen and straight sinus is seen. The majority of patients diagnosed in the neonatal period present with life-threatening, intractable cardiac decompensation and high-flow arteriovenous fistulae, CHF due to AV shunting (L -> R) and predominantly RIGHT-sided cardiomegaly, a wide superior mediastinum and retrosternal fullness. In neonates it causes high output congestive heart failure. In older children it obstructs the Aqueduct of Sylvius,producing non communicating hydrocephalus. QUESTION NUMBER 1083 An 8 year old boy with mild intellectial impairment has frequent episodes of hemiparesis and transient ischemic attacks precipitated by physical exercise. He undergoes cerebral angiography which showed occlusion of the supra-clinoid portion of both internal carotid arteries with perforator branches arising from the circle of Willis. The patient's dominant cerebral blood supply was via the left vertebral artery. A likely diagnosis is a) Moya Moya disease b) child abuse c) migraine d) Takayasu's disease e) disseminated gonorrhea Correct answer: A EXPLANATION Moya Moya disease is a rare disease causing occlusion of the supraclinoid portions of the internal carotid Arteries There are no specific symptoms or signs related to moyamoya syndrome. The various clinical manifestations are generally caused by cerebrovascular ischemic or hemorrhagic events. Headaches and seizures are also seen. Clinically the disorder presents in children with transient ischemic attacks (TIA) frequently with episodes of hemiparesis or other focal neurological signs, often precipitated by physical exercise or hyperventilation. There may be a more chronic course of worsening with a gradual impairement of intellectual deterioration. Cerebral angiography is the cornerstone of the diagnosis of moyamoya syndrome. The characteristic angiographic findings of moyamoya syndrome are a symmetrical stenosis (tapering) or occlusion of the intracranial internal carotid artery, as well as the origin of the anterior and middle cerebral artery associated with an enlargement of the basal penetrating branches of these arteries in an apparent attempt to provide collateral circulation and giving the classic cloud of smoke appearance QUESTION NUMBER 1130 The teacher of a 6 year old boy has complained to his parents that he is disrupting his class mates. He is

40

Abdulrahman Bashire 41 Children hospital Benghazi inattentive in class and interupts her constantly.Also, his attention span is very limited. She is worried that he may fall or accidentally hurt himself in the playground because he is so inattentive. His parents take him to the general practitioner who ,after examining the child, decides the following is the most appropriate medication a) haloperidol b) Amitriptiline c) methylphenidate d) diazepam e) imipramine Correct answer: C EXPLANATION Methylphenidate is an orally administered central nervous system stimulant that is chemically and pharmacologically similar to the amphetamines In treating children with attention-deficit hyperactivity disorder, methylphenidate produces a calming effect. This action of methylphenidate results in a decrease in hyperactivity and an increase in the child's attention span QUESTION NUMBER 4812 Progression to the tubrous nodules in tuberous sclerosis has been associated with a) mutation of TSC1 and TSC2 genes b) mutation of DPC4 gene c) Mutation of p21 gene d) Mutation of Rb gene e) Mutation of p53 gene Correct answer: A EXPLANATION Two loci for tuberous sclerosis have been found: TSC1 on chromosome 9, and TSC2 on chromosome 16. It took four years to pin down a specific gene from the TSC1 region of chromosome 9: in 1997, a promising candidate was found. Called hamartin by the discoverers, it is similar to a yeast protein of unknown function, and appears to act as a tumor suppressor: without TSC1, growth of cells proceeds in an unregulated fashion, resulting in tumor formation. TSC2 codes for a protein called tuberin, which, through database searches, was found to have a region of homology to a protein found in pathways that regulate the cell (GAP3, a GTPase-activation protein). QUESTION NUMBER 1176 A 2 year old boy is brought to the pediatrician by his father after two days of persistent fever and irritability. The child has been eating somewhat poorly for the past two days. The child is sleeping in his father's arms but is easily aroused by verbal stimuli. A few macular, red, discrete lesions are seen on his chest and abdomen. His right tympanic membrane is slightly red. His blood pressure is 82/ 48, pulse rate 144 and respiratory rate 36. His rectal temperature is 38.4C. The pediatrician proceeds to take blood tests. He notes that while taking the bloods the child, although distressed, cried minimally and resisted less than would be anticipated. The blood results are returned an hour hour later and are unremarkable except for a white blood count of 15,200 .The pediatrician decides to perform a lumbar puncture. A gram stain of the CSF reveals gram-negative, bean-shaped diplococci. This boy has been sharing a cot with his twin brother called John. The father is concerned that John may alsobecome ill John should be given prophylaxis with: a) rifampicin b) sulfonamide c) penicillin<br> d) chloramphenicol e) a polysaccharide vaccine Correct answer: A 41

Abdulrahman Bashire 42 Children hospital Benghazi EXPLANATION <a href="http://www.mcqs.com/realaudio/step2/1/1.ram">EXPLANATION IN REAL AUDIO</a> An interesting fact in this case is that here we are dealing with a child who does not have very obvious signs of meningitis. However, he does in fact have certain worrying features which arouse the suspicion of meningitis. The first is the rash. Another worrying fact is that he remains drowsy and does not react adversely when the physician takes a blood sample, as a child he would be expected to, have to having his bloods taken. These suspicions prompted the phsiscian to take a CSF sample. Here we find that this revealead the gram negative diplococci - and so this brings us to the diagnosis of meningitis. Neisseria species are gram-negative diplococci. There are two pathogenic and many commensal species. The pathogenic species are N. meningitidis (meningococcus) and N. gonorrhoeae (gonococcus). The organisms are gram-negative, bean-shaped diplococci that typically appear in pairs with the opposing sides flattened. They are readily phagocytosed by neutrophils. They are nonmotile and non-acid fast. Capsules and pili may be demonstrated. Close contacts of a patient who has meningococcal disease include a) household members, b) day care center contacts, and c) persons directly exposed to the patient's oral secretions (e.g., through mouth-to-mouth resuscitation or kissing) (3). Primary, Secondary, and Co-Primary Cases A primary case is a case that occurs in the absence of previous known close contact with another casepatient. A secondary case is defined as one that occurs among close contacts of a primary case-patient greater than or equal to 24 hours after onset of illness in the primary case-patient. If two or more cases occur among a group of close contacts with onset of illnesses separated by less than 24 hours, these cases are considered to be co-primary. Standard recommendations dictate the administration of rifampin or a quinilone anti-infective agent to those who resuscitate, intubate, suction or otherwise have contact with the oral secretions of a patient with Meningococcemia Rifampin must be used for sulfonamide-resistant strains or when the susceptibility is unknown. Sulfonamides should only be used if there is known susceptibility. Penicillin cannot be used because it does not penetrate uninflamed nasopharyngeal mucosa. QUESTION NUMBER 857 A 10 year old girl with mild mental retardation is known to have tuberous sclerosis. She is complaining of a recent onset of headaches together with severe vomiting which are waking her up in the middle of the night. Which of the following pathologies would you most likely suspect. a) meningioma b) optic nerve glioma c) giant cell astrocytoma d) metastatic neuroblastoma e) choroid plexus papilloma Correct answer: C EXPLANATION In patients with tuberous sclerosis, subependymal giant cell astrocytoma is common and discovered in childhood. Optic nerve gliomas may be found in such patients however the incidence is not so common. QUESTION NUMBER 1082 A 7 year old child with a seven day history of acute sinusitis tenderness becomes rapidly unconcious and has repeated seizures within 24 hours. Examination reveals several diffuse focal neurologic deficits. An MRI examination of the brain suggesets edematous and hyperemic gyri with small foci of infarction. the likely pathology is a) encephalitis b) child abuse c) meningitis d) thrombosis of a major dural sinus e) pseudotumour cerebri Correct answer: D EXPLANATION 42

Abdulrahman Bashire 43 Children hospital Benghazi Intracranial extension of infection is the second most common complication of acute sinusitis and this may cause thrombosis of a major dural sinus . Infection may enter the intracranial compartment by two routes. Direct extension may occur through necrotic areas of osteomyelitis in the posterior wall of the frontal sinus or roof of the sphenoid sinus. The underlying dura becomes thickened with inflammatory exudate, and a heavy layer of granulation tissue develops. Bacterial penetration may take place along the course of the small vessels that traverse the dura. This results in a subdural empyema. This direct route of intracranial extension is more commonly associated with chronic otitis rather than sinusitis. An alternative route of intracranial bacterial entry is provided by the valveless venous network which interconnects the intracranial venous system and the vasculature of the sinus mucosa. Thrombophlebitis originating in the mucosal veins progressively involves the emissary veins of the skull, the dural venous sinuses, the subdural veins, and finally the cerebral veins. By this mode of spread, there may be no involvement of intermediary structures and therefore may be no evidence of extradural pus or osteomyelitis. Further intracranial spread of infection depends on the competence of the arachnoid as a barrier to bacterial invasion. Although subarachnoid infection secondary to subdural empyema is uncommon in young children, extensive cortical thrombophlebitis is a frequent complication. The involved gyri are edematous and hyperemic, often showing small foci of infarction. Sometimes septic thrombosis of a major dural sinus occurs, resulting in massive bilateral cerebral edema and hemorrhagic infarction. This is often accompanied by the appearance of seizures focal neurologic deficits, and increased intracranial pressure. QUESTION NUMBER 914 The teacher of a 6 year old boy has complained to his parents that he is disrupting his class mates. He is inattentive in class and interupts her constantly. Also, his attention span is very limited. She is worried that he may fall or accidentally hurt himself in the playground because he is so inattentive. His parents take him to the general practitioner who, after examining the child, decides on medication that is likely to a) inhibits the release of dopamine b) inhibit the release of norepinephrine c) inhibit the reuptake of catecholamines d) destroy monoamine neurotransmitters e) increase the activity of monoamine oxidase Correct answer: C EXPLANATION These clinical features are suggestive of Attention-deficit hyperactivity disorder. This disorder is typically treated with stimulant medications such as methylphenidate and amphetamines. These drugs release and inhibit the reuptake of catecholamines, mainly dopamine, in the central nervous system. The clinical effects peak at about 1 h after each dose and dissipate in about 4 h so the drugs are usually given two or three times per day. Psychomotor stimulants produce their characteristic behavioral effects by increasing synaptic activity of the monoamine neurotransmitters, dopamine, norepinephrine and serotonin. They are called indirect agonists because their primary effect is to increase the ability of the neurotransmitters to act, without having a direct effect on the postsynaptic receptors for these neurotransmitters. Although producing slightly different cellular and molecular effects, the final outcome for each drug in this class, an increase in monoamine activity, is quite similar. Amphetamine increases the activity of monoamines in several important ways: 1) Amphetamine stimulates the release of dopamine and norepinephrine from catecholamine nerve terminals, increasing the amount of these neurotransmitters in the synapse. 2) Amphetamine also inhibits reuptake of the catecholamines, increasing their ability to activate receptors. 3) In addition, amphetamine inhibits monoamine oxidase, the enzyme responsible for the destruction of monoamine neurotransmitters, further increasing the availability of these neurotransmitters. 4) Finally, there is some evidence that amphetamine may directly activate catecholamine receptors, further 43

Abdulrahman Bashire 44 Children hospital Benghazi contributing to monoaminergic activity. Since a diagnosis of ADHD or HKD usually results in a trial of stimulant medication, the large differences in administrative prevalence between countries are reflected in prescriptions of this treatment. Many controlled trials with thousands of accumulated patients and the views expressed in thousands of clinical and empirical reports have established a consensus about stimulant medications. About 80% of patients have clinically meaningful benefits and despite some common side-effects (sympathomimetic activity inhibits appetite and sleep and dopaminergic effects may elicit tics) the medications are safe. Each dose of the stimulant medication has a short duration of action with little carry-over of effects; long-term tolerance is rare since continual increases in dose are not needed to maintain clinical benefit; and adults with ADHD/HKD also benefit from this treatment. The clinical response of ADHD/HKD patients is not considered to be a paradoxical response, since children and adults without this disorder, even though they are not impaired in the domains of functioning affected, respond to the same doses of stimulants in the same direction (ie, decreased activity). Methylphenidate is an orally administered central nervous system stimulant that is chemically and pharmacologically similar to the amphetamines. In treating children with attention-deficit hyperactivity disorder, methylphenidate produces a calming effect. This action of methylphenidate results in a decrease in hyperactivity and an increase in the child's attention span. QUESTION NUMBER 1079 2 month old female is brought to the Casualty department with acute onset of loss of consciousness. On examination the child is unresponsive. A noncontrast CT of the head demonstrates an extensive area of edema involving the right cerebral hemisphere as well as the ACA territory of the left cerebral hemisphere. The bone windows also demonstrate a subdural hematoma. These features are suspicious of a) Moya Moya disease b) child abuse c) medulloblastoma d) aneurysmally dilated great vein of Galen e) thrombosis of a major dural sinus Correct answer: B EXPLANATION Cerebral infarction without an underlying condition or trauma in childhood is rare. The finding of an associated subdural hematoma suggests trauma or child abuse. The battered child syndrome is the most common cause of serious intracranial injuries in children less than 1 year of age, and it is the 3rd most common cause of death in children after SIDS and true accidents. CNS findings include the following:Impact injury:Skull fracture Subdural hematoma Brain contusion Cerebral hemorrhage Infarction or generalized edema. QUESTION NUMBER 5985 The most useful medication that may be used in juvenile myoclonic epilepsy a) phenytoin b) primidone c) carbamazepine d) valproic acid e) Phenobarbital Correct answer: D EXPLANATION Valproic acid is a drug of choice for: Generalized epilepsy of unknown cause that produces more than one type of seizure. Absence seizures. Ethosuximide is the other drug of choice. If a person has absence seizures as well as other 44

Abdulrahman Bashire 45 Children hospital Benghazi types of seizures, however, valproic acid usually works better because it can control several types of seizures. Myoclonic seizures, such as those caused by juvenile myoclonic epilepsy. Clonazepam may be used when valproic acid does not work. Valproic acid is also considered a first-line drug for treating partial seizures.

45

Das könnte Ihnen auch gefallen